Afar Answerkey Synthesis
Afar Answerkey Synthesis
ANSWER KEY
FOR
ADVANCED
FINANCIAL
ACCOUNTING AND
REPORTING
(Compilation of Test banks, Reviewers,
Quizzers etc.)
5TH YR BS-ACCOUNTANCY
(2018-2019)
1
TABLE OF CONTENTS
1.0 PARTNERSHIP ACCOUNTING
1.1 Formation 9
1.2 Operations 13
1.3.1.2 By investment 30
1.4 Liquidation 42
2
4.2 Long – term Construction Contracts (PAS 11 / PFRS 15)
4.2.1 Journal entries and determination of revenue, costs and gross profit
contract work
4.3.1 Journal entries and determination of revenue, costs and gross profit
3
5.0 ACCOUNTING FOR HOME OFFICE, BRANCH AND AGENCY
5.1 Transactions on the books of the home office and the branch 131
4
8.1.3 Determination of:
5
TRANSLATION OF FOREIGN CURRENCY FINANCIAL
10.0 STATEMENTS (PAS 21/ PAS 29)
Translation from the Functional Currency to the Presentation Currency 274
10.1 (Closing/ Current Rate Method)
13.2 Accounting for build, operate & transfer (PFRIC 12) 320
6
14.0 COST ACCOUNTING
14.1 System of cost Accumulation or Costing System
7
14.6 Activity – based costing system (ABC costing)
8
PARTNERSHIP ACCOUNTING
Formation
1. C.
Goodwill of Rose, Violet, Patrick and Leo 30,000
Goodwill of Edgar and Fint 10,000
Total 40,000
2. C.
Rose
Unadjusted Capital 50,000
Share in undervalued assets (20,000 x 40%) 8,000
Share in goodwill (30,000 x 40%) 12,000
Adjusted Capital 70,000
3. C.
Rose [50,000 + (2,400 x 40%)] 50,960
Edgar [60,000 - (2,400 x 40%)] 58,800
(20,000+30,000)
68,000 68,000
4. D.
5. D.
6. A.
7. A.
8. A.
9. A.
10.C.
11.C.
9
12.A.
Adams Baker
Merchandise 0 320,000
Building 0 840,000
13.C.
14.D.
15.A.
16.B.
17.A
18.B.
19.D.
20.C.
Agreed capital 10,000,000
25%
Capital credited to Jessica 3,000,000
Contributed capital of Jessica (800,000)
Additional contribution 2,200,000
21.C.
Robert Sean
Building 600,000
282,500 525,000
22.C.
3/1 140,000 x 4/10 56,000
7/1 180,000 x 1/10 18,000
10
8/1 165,000 x 5/10 82,500
Average Capital 156,500
Multiplied by 10%
Annual interest 15,650
Multiplied by 10/12
Prorated interest for the year 13,042
23.C.
24.A.
25.D.
26.C.
27.A
28.D.
29.C.
30.C.
31.B.
A’s contribution (190,000 – 60,000) 130,000
B’s contribution 100,000
Total capital contribution 230,000
Capital interest of A 60%
Capital credited to A 138,000
Contribution of A 130,000
Bonus to A 8,000
32.C.
Magic = 596,250 – 5,550 = 590,700
Moto = 335,000 – 4,050 – 9,000 = 321,950
33.A.
Magic = 650,000 - 590,700 = 59,300
Moto = 400,000 - 321,950 = 78,050
34.A.
Magic 590,700 Total Capital 921,650
35.A.
Total Agreed Capital of HAyaan Mo (300,000 x 50%) 150,000
Total Contributed Capital of HAyaan Mo 90,000
Cash to be contributed 60,000
36.D.
Total Liabilities 25,000
Total Capital 300,000
11
Total Assets 325,000
37.A.
Required capital of Petmalu (287,500 x 60%) 172,500
Non-cash assets contribution of Petmalu (125,000 – 30,000) 95,000
38.C.
Contribution of Gandara 115,000
Contribution of Petmalu (125,000 – 30,000 + 50,000) 145,000
Total partnership Capital 260,000
39.B.
Total Contributed Capital 210,000
Divided by 2
Petmalu’s Capital 105,000
40.C.
Total Contributed Capital 184,000
Divided by 2
BInos,Capital 92,000
12
PARTNERSHIP ACCOUNTING
OPERATIONS
1. A.
Net income P 25,000
Accrued expenses 1,200
Inventory overstated (3,100)
Purchases not recorded (2,000)
Prepaid income 1,500
Unused supplies 900
Total P 23,500
Multiply by 40%
Share in net income of Jimmy P 9,400
2. A
3. B. P290,000
40,000 = 25,000 + 10% (NI – B - S)
40,000 = 25,000 + 10% (NI – 15,000 – 125,000)
40,000 = 25,000 + .1NI – 14,000
29,000 = .1NI
290,000 = NI
4. D.
Gross income (150,000 – 40,000 + 10,000) 120,000
Expenses 20,000
Net income 100,000
13
7. A
Average capital
DIAMOND RUBY
6% 6%
Interest 3,123 Interest 7,220
14
9. D.
Statement of Capital:
Arthur Baxter Cartwright Total
Beginning balance P60,000 P80,000 P100,000 P240,000
Net income 7,800 29,800 12,400 50,000
Drawing (given) ( 5,000) ( 5,000) ( 5,000) ( 15,000)
Ending capital P62,800 P104,800 P107,400 P275,000
10.C.
A B C TOTAL
Bonus* P 4,000 P 4,000
Interest** 1,000 1,000
Salaries 10,000 12,000 22,000
Balances (4:4:2) 6,800 6,800 3,400 17,000
P19,400 P44,000
=P4,400 - .10B
1.10B = 4,400
B = P4,400
11.A P9,400
Net income per book P25,000
Add (deduct): Adjustments\
Accrued expenses not recorded in 2010 1,200
Inventory overstated in 2011 ( 3,100)
15
Purchases not recorded in 2011 ( 2,000)
Income received in advanced not adjusted in 2010 1,500
Unused supplies not taken up in 2011 900
Adjusted net income P23,500
Multiply by profit and loss ratio 40%
Profit share of Rosa P 9,400
12.A.
Ryan Sandra Total
Salary for 2014 18,000 18,000
Balance of 2014 income
(25,000 – 18,000 – 4,000) 1,500 1,500 3,000
2013 Adjustment (60:40) 2,400 1,600 4,000
Share In net income 21,900 3,100 25,000
13.B.
Daniel Barry Total
Salaries (10 months) 8,000 10,000 18,000
Interest* 2,800 3,600 6,400
Remaining ( 200) ( 200) ( 400)
14.A.
Kimberly Ericka Julie Grace Total
Bonus
Total of salaries, interest and balance 220,000
Less: interest of Julie ( 17,500)
Balance net of bonus 202,500
Divided by 0.80
Multiply by bonus 0.20
Bonus 50,625
Bonus
17
Multiply by bonus 0.20
Bonus 9,375
17. B.
To equate P40,000 to P25,000 plus bonus. The bonus should amount to
P15,000 (P40,000 – P25,000). Based on the foregoing the following should be
developed;
Bonus = 10% (NI – Salaries – Bonus)
P15,000 = .10 [NI – (P100,000 + P25,000) – P15,000]
P15,000 = .10 (NI – P140,000)
P15,000 = .10NI – P14,000
P29,000 / .10= NI
NI = P290,000
Or, Alternatively
P40,000 = P25,000 + .10 (NI – salaries – bonus)
P40,000 = P25,000 + .10 [NI – (P100,000 + P25,000) –
P15,000]
P40,000 = P25,000 + .10 (NI – P140,000)
P40,000 = P11,000 + .10NI – P14,000
P29,000 = .10NI
NI = P290,000
18.B.
19.B.
Bryant Milton Pine Total
20.A.
B = 20% (NI – I – S – B)
B = 20% [168,000 – (8%(150,000)) – 60,000 – B]
B =20% (96,000 – B)
B = 19,200 - .20B
1.20B = 19,200
B = 16,000
18
21.B.
Stones Miles Kiney Total
22.B.
Stones Miles Kiney Total
24.A.
25.B.
Original Capital of Bea (5,000,000 x 50%) 2,500,000
Share NI,2021 (2,000,000 x 20%) 400,000
Withdrawals (300,000)
Bea Capital 12/31/21 2,600,000
Share NI, 2022(1,000,000 x 10%) (100,000)
Withdrawals (200,000)
Bea Capital 12/31/22 2,300,000
26.D.
Lebron Capital 12/31/31 6,500,000
Lebron original contribution (5,000,000)
Withdrawals 1,000,000
19
Share in NI 2,500,000
Interest (5,000,000 x 20%) (1,000,000)
Share in remainder 1,500,000
Divided by 50%
Total remainder 3,000,000
Withdrawal ( 600,000)
Total 4,400,000
29.C.
30.A.
Solution in No. 29 and 30
20
T – 36% 3,000,000 1,800,000 360,000 2,160,000
31.C.
32.A.
Net decrease in capital (120,000)
Add: Withdrawals 260,000
Total 140,000
Less: Additional investments 50,000
Profit share 90,000
Profit share percentage 30%
Total partnership net profit 300,000
33.C.
34.B.
35.C.
X Y Z TOTAL
36.B.
X Y Z TOTAL
37.B.
X Y Z TOTAL
38.C.
Mariano Lucas Total
39.A.
Total share in Capital 20,342 5,268 25,610
Add: Capital, beginning 125,000 75,000 200,000
Total 145,342 80,268 225,610
Less: Withdrawals 20,000 30,000 50,000
Ending capital 125,342 50,268 175,610
40.D.
Andal Briones Camba Total
22
23
DISSOLUTION/ CHANGES IN OWNERSHIP INTEREST
By purchase of interest
1. D.
Equity balance of E 50,000
Amount paid 46,000
Bonus to C & D 4,000
C 120,000 + (3/5 x 4,000) 122,400
D 60,000 + (2/5 x 4,000) 61,600
2. A.
Matt Tim Luke Total
Beginning Capital 45,000 30,000 50,000 125,000
Salaries (3years) 30,000 30,000 30,000 90,000
Bonus (3years) 24,000 24,000
Balance/remaining 40,000 80,000 120,000 240,000
Total 115,000 164,000 200,000
Net income
Y1 125,000
Y2 110,000
Y3 144,000
Total 379,000
Depreciation ( 25,000)
Adjusted net income 354,000
3. B.
Samuel Mona Isabela Total
5 4 1
Adjustment of assets
24
4. B.
Samuel Mona Isabel FrancoTotal
5 4 1
Adjustment
[(40,000/20%) – 150,000]/50%
5. B.
Samuel Mona Isabel FrancoTotal
5 4 1
Adjustment
[(250,000-(120,000/.25)]/.50
25
6. A.
7. C.
Dell, Capital (P425,000 x 20%) 85,000
Gore, Capital (P275,000 x 20%) 55,000
Mann, Capital 140,000
8. C.
9. D.
Kardo, capital 60,000
Siyano, capital 20,000
Total 80,000
Multiplied by 20%
Lily, capital 16,000
10.B.
Partner Sold interest Loss share Cash received
Kardo (60,000 x 20%) 12,000 (600) 11,400
Siyano (20,000 x 20%) 4,000 (400) 3,600
Total 16,000 (1,000) 15,000
11.B.
T, Capital 5,000,000
Multiply by % Purchase 40%
S, Capital 1,200,000
12.A.
13.A.
14.B
264,000 – [(278,000 + 418,000 + 192,000) x 1/5] = 86,400
15.A.
Lima 100,000 x 80% = 80,000
Mitra 50,000 x 80% = 40,000
16.A.
Asset revaluation (60,000/20%) – 150,000 150,000
Lima [100,000 + (150,000 x 75%)] 170,000
Mitra [50,000 + (150,000 x 25%)] 70,000
Nova 60,000
17.D.
Felix Elias Total
26
Net profit 5,430 10,860 16,290
18.D.
19.C.
S E C
Asset revaluation
20.D.
Total capital of new partnership (840,000/75%) 1,120,000
Multiply by % of interest 25%
Amount to be invested by X 280,000
21.C.
Juan Cosme Luna Magno
Capital Balances, April 30, 2016 360,000 225,000 135,000
1/6 interest transferred to Magno (60,000) (37,500) (22,500) 120,000
Balances 300,000 187,500 112,500 120,000
27
Cash transfers to equalize inv. (100,000) 12,500 87,500
22. B.
Agreed capital 330,000
Capital Contribution 315,000
Asset Revaluation 15,000
23. D.
80,000 + 12,000 – 70,000 = 22,000
24. C.
Capital balance before admission of Manalo 80,000
Interest sold to Manalo (80,000 x 15%) (12,000)
Share asset revaluation (15,000 x 3/10) 4,500
Share in the bonus from Manalo (22,000 x 3/10) 6,600
Capital balance after admission of Manalo 79,100
25. A.
The capital balances would be the same as the balances prior to sale of
interest.
26. B.
27. B.
28. D.
Solution for nos. 26-28000’s are omitted)
C D O X Total
Asset Revaluation 6 6 8 20
28
Capital bal. after adm. 162 162 316 160 800
29.A.
C 30% x 75% = 22.5%
D 30% x75% = 22.5%
O 40% x 75% = 30%
X 25%
30. C.
31.A.
140,000/ ¼ = 560,000 – (200,000 + 160,000 + 120,000) = 80,000
32.A.
Conde, Capital 90,000
Cuenco, Capital 60,000
Catral, Capital 150,000
33.C.
Other Assets 360,000
Conde, Capital 270,000
Cuenco, Capital 90,000
34. A.
Locsin, Capital (240,000 x ¼) 60,000
Montes, Capital (120,000 x ¼) 30,000
Nava, Capital 90,000
35. C.
Locsin, Capital (240,000 x 1/3) 80,000
Montes, Capital (120,000 x 1/3) 40,000
Nava, Capital 120,000
36. B.
Other assets 180,000
Locsin, Capital (180,000 x ¾) 135,000
29
Montes, Capital (180,000 x ¼) 45,000
30
DISSOLUTION/ CHANGES IN OWNERSHIP INTEREST
By investment
3. B. P 48,900
Old capital adjustment P 108,000
New Partner Contribution 55,000
Total 163,000
Multiply by 30%
New Partner’s Capital P 48,900
4. A.
TCC TAC
Ricahrd 200,000 (8,000)
Jason 100,000 (12,000) 88,000
Genesis 100,000 20,000 120,000
400,000 0 400,000
Bonus to Genesis = 5,000
Share of Jason in the residual profit ( 50,000x 40% ) = 20,000
5. D.
Georgina Nelson Loren Fidel
31
Balances 300,000 170,000 70,000 540,000
TCC TCA
G 300,000 (30,000) 270,000
N 170,000 (8,000) 162,000
L 70,000 38,000 108,000
F 270,000 0 270,000
810,000 0 810,000 (540,000 / 2/3)
6. C.
7. B.
8. C. P35,374 refer to no. 9
9. C. P17,687
Unadjusted capital of CC P33,000
10.D.
Rosa: 50% x 80% = 40%
Susan:30% x 80% = 24%
Tina 20% x 80% = 16%
Vida 20%
100%
32
11.C.
12.A.
Goodwill [(P300,000/25%) – P1,000,000] 200,000
Dell, Capital (P200,000 x 3/5) 120,000
Gore, Capital (P200,000 x 2/5) 80,000
Cash 300,000
Mann, Capital 300,000
13.A.
Cash 150,000
Goodwill [(P700,000 / 80%) - P850,000] 25,000
Mann, Capital 175,000
14.C.
15.A
Total agreed capital 20,000,000
Multiplied by 30%
16.B.
Kardo 60,000
Siyano 20,000
33
Lily 15,000
New total capital 95,000
Multiplied by 20%
17.C
18.C
19.B
Solution in No. 17, 18 and 19
TCC TAC Bonus Asset revaluation
A 1,000,000 1,400,000 (100,000) 500,000
B 2,500,000 3,100,000 (150,000) 750,000
C 500,000 1,500,000 (250,000) 250,000
D. 1,500,000 2,000,000 500,000 0
Total 5,500,000 8,000,000 - 2,500,000
20.C.
21.B.
22.B.
23.D.
24.B.
25.C.
26.D.
Capital of Mison Prior to the admission 105,000
Share in the bonus from Zamora (15,000 x 50%) 7,500
Capital of Mison in the new partnership 112,500
27.C.
TAC TCC Total
Z 195,000 195,000
28.D.
195,000 –(525,000 x 1/3) = 20,000
34
29.A.
Old partner’s capital contribution 600,000
Divided by % of interest of old partners 75%
Total agreed capital 800,000
Multiply by % of interest of new partner 25%
Capital credited of Sison 200,000
Bonus to Sison (70,000)
Cash to be contributed by Sison 130,000
30.B.
Juan Cosme Luna Magno
31.C.
Agreed Capital (201,650 + 201,150 + 201,650) ÷ ¾ 805,933
Interest of Magno ¼
Required Capital Credit to Magno 201,483
Capital Balance of Magno before investing 121,150
Cash to be invested by Magno 80,333
32.A.
Galang Hizon Istela
Asset revaluation method:
Capital contribution of partners 600,000 480,000 500,000
Asset revaluation 252,000 168,000
Additional depreciation (140,000) (140,000) (140,000)
Capital balances 712,000 508,000 360,000
35
Bonus method:
33. A.
Campos Centeno
34.C.
Total Capital (614,476 + 683,052) 1,297,528
Total Liabilities 967,590
Total Assets 2,265,118
35. D.
Total Capital (1,297,528/80%) 1,621,910
Multiply by: interest of Coronel 20%
Contribution of Coronel 324,382
36
36. D.
Campos Centeno
37. C.
Campos Centeno Coronel
38. A.
Total capital (3,000,000/ 80%) 3,750,000
Capital interest of Florez 20%
39. B
40. C.
37
DISSOLUTION /CHANGES IN OWNERSHIP INTEREST
Withdrawal, retirement or death of a partner
1. C
2. B
3. A
4. A (P4,000 x 2/5 = P1,600)
5. D (P3,000 / 40% = P7,500)
6. A (P12,000/3 = P4,000)
7. C
8. A
15. A
16. A
17. B
18. A
19. D
Solutions for nos. 16-19
16. Bonus Method
39
18. The bonus method will be preferred by Manzano
19.
PARTNERSHIP ACCOUNTING
Liquidation
1. A
Noncash assets 2,000,000
Sale price 1,500,000
Loss on sale 500,000
Liquidation expenses 100,000
Total loss on liquidation 600,000
Capital of A 700,000
Receivable from A (500,000)
2. C Net capital of A 200,000
Share in total loss (60% x 600,000) (360,000)
Debit balance in capital of A (160,000)
3. B
Capital Balance of B before liquidation ( 650,000)
Add: Payable to B 1,000,000
Capital Balance of B after the right of offset 350,000
Less: Share of B in Total Loss on Liquidation (600,000x 10%) 60,000
42
Capital Balance of B after loss on liquidation but before absorption 290,000
of A’s insolvency
Less: Share of B in A’s debit capital balance (P160,000 x 1/4) (40,000)
Cash received by B at the end of partnership liquidation 250,000
Capital Balance of C before liquidation 350,000
Add: Payable to C 100,000
Capital Balance of C after the right of offset 450,000
Less: Share of C in Total Loss on Liquidation (600,000 x 180,000
30%)
Capital Balance of C after loss on liquidation but before 270,000
absorption of A’s insolvency
Less: Share of C in A’s debit capital balance (P160,000 x (120,000)
3/4)
Cash received by C at the end of partnership 150,000
liquidation
4. B
5. D
Capital Balance of C before liquidation 400,000
Less: Share of C in Total Loss in Liquidation during January (P500,000* x (100,000)
20%)
Capital Balance of C after loss on liquidation but before absorption of A’s 300,000
insolvency
Less: Share of C in A’s debit balance (P150,000 x 2/5) (60,000)
Cash received by C at the end of partnership liquidation 240,000
43
Add: Net proceeds from sale of noncash asset during January (1,000,000 + 1,100,000
100,000)
Less: Cash paid for liquidation expenses during January (50,000)
Less: Cash paid for liabilities to third person during January (P2,000,000 x (400,000)
20%)
Less: Cash withheld for unpaid liabilities to third persons (P2,000,000 x 80%) (1,600,000)
Less: Cash withheld for estimated future liquidating expenses (150,000)
Cash available for distribution to partners 500,000
Less: Total capital of all partners (100,000+500,000+400,000) (1,000,000)
Total loss on liquidation for the first month of installment 500,000*
6. B
Capital of A before liquidation 100,000
Share in loss on liquidation (50% x 500,000) (250,000)
Debit balance in capital of A (150,000)
7. D
Estimated future liquidating expenses on January 31, 2019 150,000
Add: carrying amount of remaining noncash assets on January 31,
2019 400,000
(1,400,000 – 1,000,000)
Maximum possible loss on January 31, 2019 550,000
B’s share in maximum possible loss (P550,000 x 30%) 165,000
8. C
9. C
10. A
11. A
12. B
13. D
14. C
15. C
16. C
SOLUTION: (13-16)
44
Beg 55,000 85,000 65,000 75,000 280,000
225,000
175,000
112,500
Priority 3
Liabilities (220,000)
45,000 105,000
2.
Liabilities 220,000
45
3.
2nd Distribution
4.
17. A
18. A
19. D
20. B
21. A
22. C
23. B
24. C
25. D
26. B
27. C
46
28. A
29. A
30. C
31. A Aguilar Benito Casimiro David
32. B Capital balances before liquidation P 11,000 P 10,300 P 13,700 P 9,000
Loan from partners 2,000
Total partners’ interest P 13,000 P 10,300 P 13,700 P 9,000
Loss on realization (P46,000 – (13,600) ( 10,200) ( 6,800) ( 3,400)
P12,000)
Balances P( 600) P 100 P 6,900 P 5,600
Additional loss to partners 600 ( 300) ( 200) ( 100)
Balances -------- P( 200) P 6,700 P 5,500
Additional loss to partners 200 ( 133) ( 67)
Distribution of cash to partners --------- --------- 6,567 5,433
33. A
34. B
Non-cash C A P I T A L
Cash Assets Liabilities Elma Erica Edna
Profit and loss ratio 3/8 3/8 2/8
Balances before P 80,000 P810,000 P270,000 P60,000 P290,000 P270,000
liquidation
Sale of non-cash 634,000 (810,000) (66,000) ( 66,000) ( 44,000
assets and )
distribution of loss
47
Payment of ( 24,000) ( 9,000) ( 9,000) ( 6,000
liquidation expenses )
Balances P690,000 P270,000 (P15,000) P215,000 P220,000
Payment of liabilities (270,000) (270,000)
Balances P420,000 (P15,000) P215,000 P220,000
Additional investment 15,000 15,000
of Elma
Balances P435,000 P215,000 P220,000
Payment to partners ( 435,000) ( 215,000) ( 220,000)
35. A
36. A
37. B
48
Balances P690,000 P70,000 P200,000 (P15,000) P215,000 P220,000
38. A
39. B
40. C
NR from Non-cash NP C A P I T A L
Cash Erica Assets to Elma Liabilities Elma Erica Edna
Profit and loss 3/8 3/8 2/8
ratio
Balances before P 80,000 P110,000 P700,000 P70,000 P200,000 P60,000 P290,000 P270,000
liquidation
Sale of non-cash 634,000 (700,000) (24,750) ( 24,750) ( 16,500)
assets and
distribution of
49
loss
Payment of (24,000) ( 9,000) ( 9,000) ( 6,000)
liquidation
expenses
Balances P690,000 P110,000 P70,000 P200,000 P26,250 P256,250 P247,500
Payment of (200,000) (200,000)
liabilities
Balances P490,000 P110,000 P70,000 P26,250 P256,250 P247,500
Offset of
receivable (110,000) ( 110,000)
against credit
balance in the
capital of Erica
Balances P490,000 P70,000 P26,250 P146,250 P247,500
Payment to (490,000) ( 70,000) ( 26,250) ( 146,250) ( 247,500)
partners
CORPORATE LIQUDATION
Statement of Affairs
1. A
2. C
3. B
4. A
5. A
6. B
7. A
8. A
9. C
50
Amount received by holder of accounts payable (P100,000 x 40,000
40%)
Amount
Amount received
received by holder
by partially of mortgage
secured loan payablepayable (Fair value of 100,000
340,000
Land)
Less: Fair value of collateral – machinery (300,000)
Note: Theamount
Recovered mortgage
frompayable will be fully
the unsecured collected
portion because
of partially it is fully
secured loans 40,000
secured credit.
payable
Divided by unsecured portion of partially secured loan payable (400,000 – /100,000
300,000)
Recovery percentage on unsecured credits 40%
10. A
11. C
12.D
13.B
51
14. D
15. A
16. C
17. B
18. B
19. A
20. C
21. B
22. A
Cash 100,000
Add: Free assets from fully secured mortgage payable (P120,000 – 20,00
P100,000) 0
Total Free assets for unsecured credits with priority 120,000
Amount received by employees for their salary 120,000
Note: Since only P120,000 free assets are available, it must all be given
to employees who are preferred over the government.
23. D
Fully secured Partially Unsecured
secured
Cash P124,200 P222,000 P 59,640 P 360
Inventory 53,000 375,000 79,000
Receivable 13,000
Less:
Unsecured with
priority
Trustee’s salary (9,500) 58,500
Salaries (50,000) 80,000
payable
Taxes (4,000)
Net free assets 126,700
Total 217,860
unsecured
without priority
Recovery percentage: 126,700/217,860 = 58%
24. B
52
B: P136,000 + 13,600 = 149,600; 149,600 * 25% = P37,400
C: P137,500 + 7,452 = 144,952
D: P12,220
25. C
26. A
27. B
28. C
29. A
30. A
31. A
32. B
33. D
34. A
35. C
36. B
37. B
38. A
39. C
40. A
CORPORATE LIQUIDATION
Statement of Deficiency
1. B
2. A
3. D
53
4. C
5. C
6. D
(P560,000/P800,000) = P0.70
7. B
8. C
9. C
10. B
11. C
12. D
13. D
14. D
15. C
16. D
54
17. A
18. C
19. B
20. C
21. D
22. D
23. D
24. A
25. B
Note: Only the net realizable value of collateral inventory will be received since there
is no available net free asset.
26. D
Note: The mortgage payable will be fully collected because it is fully secured credit.
27. C
Cash P100,000
55
Amount received by employees for their salary P120,000
Note: Since only P120,000 free assets are available, it must all be given to employees
who are preferred over the government.
28. C
29. A
30. C
56
Partially secured liabilities P223,500
31. C
57
With priority
Notes payable
32. A
Notes payable
58
Total amount unpaid P969,000
33. C
34. C
35. A
36. A
37. B
38. C
39. D
40. C
59
CORPORATE LIQUIDATION
1. C
2. A
3. C
4. A
5. A
6. B
7. C
8. B
9. A
10. C
11. C
12. D
Receivable 13,000
Taxes (4,000)
60
Recovery percentage: 126,700/217,860 = 58%
13. B
D: P12,220
14. A
15. A
16. C
61
Total paid to partially secured liabilities P144,000
17. B
Cash P3,774,000
62
18. A
19. C
20. B
21. D
22. A
23. C
63
24. A
25. B
26. A
5,200,000 3,800,000
1,800,000 2,300,000
2,500,000 4,200,000
1,900,000 900,000
850,000 600,000
12,250,000 12,200,000
50,000
64
27. C
1,375,000 1,200,000
750,000 1,375,000
1,875,000 2,250,000
1,700,000 1,1625,000
3,125,000 2,800,000
8,925,000 9,250,000
425,000
28. C
29. C
Unsecured amounts:
65
Less unsecured portion 50,000 x 100% = P50,000
30. A
Cash P120,000
Total P60,000
Total P30,000
31. C
66
32. A
33. B
34. A
35. D
36. B
37. B
38. D
39. D
40. B
67
JOINT ARRANGEMENTS (PFRS 11)
Joint Operations
1. C
2. C
3. A
4. C
5. A
6. B
7. D
8. B
* The entry in the records of joint operator AA (60%) in relation to Plant assets is:
?? ??
?? ??
9. C
* The entry in the records of joint operator BB (40%) in relation to Plant assets is:
68
Gain on provision of services [60% x (P80,000-P65,000)] P9,000
10. A
JO A B
Expenses 8
1,800
42,600 77,000
34,400
11. C
12. C
13. B
14. A
Multiply by 1/3
15. B
Multiply by 1/3
69
Same amount (if P185,000 x 90%) x 1/3 P55,500
16. C
17. C
Multiply by 1/3
Multiply by 1/3
18. D
19. B
Unsold portion:
Multiply by 50%
20. B
21. A
70
Accounts payable (P4,000,000/2) P2,000,000
22. A
23. A
24. C
25. C
26. B
27. A
28. C
71
(P2,000,000 x 40%) (800,000)
29. A
30. D
31. A
Multiply by 1/3
32. C
33. C
72
Dividends received (P4M x 40%) (1,600,000)
34. C
35. B
36. C
37. B
38. A
39. C
40. A
73
JOINT ARRANGEMENTS (PFRS 11)
1. B
2. C
3. B
4. B
5. A
6. B
7. B
30% X P100,000 loss incurred by entity Z for the year ended December 31,2011
= P30,000
8. B
9. C
10. B
74
11. A
12. C
13. B
14. A
15. C
16. B
17. A
18. D
75
Cash held by BEATRIZ (P30,000 + P54,900 – P30,000 – P3,900) 51,000
19. C
At December 31, 2012 each venture must measure its investment in entity M at P0
computed as follows:
In 2012 each venturer does not recognize (P180,000 of its share of entity M’s losses.
The loss recognized by the entity is limited to its investment of P100,000)
20. A
At December 31, 2013 entities A and B must each measure their investment in entity
M at P160,000 computed as follows:
21. C
Investment in JO Tan
76
22. B
23. C
P/L P14,416
24. C
MARIBEL OLGA
22,320 42,320
15,200
33,200
18,000 20,000
25. B
26. D
Maribel Olga
Share in Profits
77
Cash on hand
Sales P75,520
Less: COS
27. B
P/L P15,700
28. C
29. B
Investment in JV P400,000
Cash P400,000
30. A
At December 31, 2013 entity M would report its investment in entity Z at P369,000
computed as follows:
78
Dividend income (30% x 150,000) (45,000)
31. D
32. B
33. A
34. C
35. B
100(6) (5) 60
Distribution of profit:
X Y Z TOTAL
79
(3) Permit fee (7) Unsold merchandise
36. A
At December 31, 2013 entities A and b must each report its investment in entity Z at
P285,000 computed as follows:
37. A
80
38. B
39. B
40. D
81
JOINT ARRANGEMENTS (PFRS 11)
1. D
2. C
3. A
4. A
5. B
6. C
7. A
8.D
Note: There is no impairment loss because fair value less cost to sell of P560,000 is
higher than carrying amount.
82
9. A
P 210,000
10. B
11. D
12. A
13. C
14. C
15. A
16. C
17. D
18. C
19. D
20. A
21. C
83
22. C
23. D
24. A
25. B
26. D
27. C
28. B
29. D
2011: CU98,000 because recoverable amount—fair value less costs to sell (CU98,000) is
less than cost (CU101,000).
30. D
31. C
32. D
33. A
The venture is a joint venture (jointly controlled entity). In accordance with their
respective accounting policies for jointly controlled entities the parties recognize their
interest in the venture using either:
84
34. B
Additional information:
The parties retain their right to the economic benefits generated from the mineral rights
—the benefits (usually received in the form of minerals) are directly related to the
amount of mineral reserves contributed by each party to the contractual arrangement.
The parties have joint and several liabilities for obligations such as decommissioning,
and also have obligations to reimburse their share of the costs incurred by the operator.
Each party has rights to its share of the joint production equipment and other resources
by directing the use of the equipment for the extraction of minerals. That share of the
equipment and resources is equivalent to each party’s mineral rights as a proportion of
the total mineral rights of the field. Put another way, each party receives benefits from
the assets in proportion to that party’s mineral rights relative to the mineral rights of
the combined field.
35. B
36. A
1 January 2011
Cash P53,000
37. B
Cash P530
85
To recognize the transaction costs incurred to acquire the investments in jointly
controlled entities.
38. C
Assessing whether a venture is governed under joint control among its parties or
whether it is controlled unilaterally by one of its parties is a matter of judgement. As a
result of this assessment, a party may conclude that:
the venture is governed under joint control (i.e. the venture is a joint
venture)
it controls the venture (i.e. the venture is a subsidiary accounted for in
accordance with Section 9 Consolidated and Separate Financial Statements)
it is an investor to the venture. (If the investor has significant influence
(18) then the venture is an associate accounted for in accordance with Section
14 Investments in Associates. If it is determined that the investor does not have
significant influence, then the investment is a financial asset accounted for in
accordance with Section 11 Basic Financial Instruments.)
39. C
Investments in jointly controlled entities are accounted for using the equity method.
The carrying amount of the investment in joint venture is calculated at cost
plus the entity’s subsequent share of the joint venture’s comprehensive
income. If at the end of a reporting period there is an indication that an investment in
a jointly controlled entity may be impaired, the entire carrying amount of the
investment is tested for impairment. If the carrying amount of the investment is found
to be less than its recoverable amount, the carrying amount is reduced to its
recoverable amount and an impairment loss is immediately recognized in profit or loss.
40. D
An investor in a joint venture that does not have joint control shall account for that
investment in accordance with Section 11 or, if it has significant influence in the joint
venture, in accordance with Section 14 Investments in Associates.
86
REVENUE RECOGNITION
Installment Sales- Recognition of gross profit- regular sales versus
installment sales
1. A
2. B
3. B
4. A
2016 2015 2014
Installment Sales
8,765,625/ 68% P12,890,625
Inst. Rec. beg. P8,387,500 P1,512,500
Inst. Rec. end (9,728,125) (3,025,000)
Collections P 3,162,500 P5,362,500 P1,512,500 (2) P10,037,500
GPR x 32% x 30% x 28%
Realized GP P 1,012,000 (3) P1,608,750 P 423,500 (1) P3,044,250
5. D
2015 installment accounts receivable P 16,250
Multiply by x 30/130 P 3,750
2015 installment accounts receivable P 90,000
Multiply by x 33 1/3 /133 1/3 22,500
Deferred gross profit P26,250
6. A
Downpayment (P545,000 x .2) P109,000
Inst. Collections (P545,000 x 8 x .40) 174,400
Collections in year 1 on Year 1 Inst. Sales P283,400
Multiply by 35/135
Realized gross profit for Year 1 P 73,474
7. A
Installment sales - Year 2 P785,000
Less Collections in Year 2
DP (P785,000 x .20) P157,000
Installment collection (P785,000 x .80 x .40) 251,200 (408,200)
87
Balance, end of Year 2 P376,800
Multiply by 35/135
Unrealized GP on Year 2 installment sales at the end of Year 2 P 97,689
8. A
Total installment accounts receivable, end of Year 3 P621,640
Multipy by 35/135
Total unrealized gross profit at end of Year 3 P161,166
9. A
Installment Sales, 2015 P25,000
Multiply by GPR 100% - (31,250/62,500) 50% P12,500
Installment Sales, 2016 P62,500
Muliply by 100% - (45,000/100,000) 55% 34,375
Realized Gross Profit P 46,875
10. C
11. D
Installment Accounts Receivable, 2016 P306,520
Multiply by 40%
Unrealized gross profit on 2016 P122,608
12. B
2006 sales - (P17,400 x 36%) P 6,264
2007 sales - P(205,400 - P200 - P25,800) x 39% 69,966
Total gross profit realized in 2016 P76,230
13. B
Installment Contracts Receivable, Jan. 1, 2015 P 344,460
Installment Contracts Receivable, Dec. 31, 2015 ( 67,440)
Repossessed refrigerator P5,400
Collections prior to default (3,200) ( 2,200)
Total P 274,820
Multiply by 34%
Total gross profit on 2015 from collections in 2016 P93,438.80
88
14. B
Installment sales P 400,000
Accounts receivable, December 31, 2011 ( 320,000)
Collections P 80,000
Multply by gross margin based on cost 66.66%
Total P 53,328
Divided by sales rate based on cost 166.66%
Realized gross profit 32,000
Installment sales P400,000
Cost of goods sold (400,000x166.66%) (240,000)
Unrealized gross profit, Jan. 1 2011 P160,000
Realized gross profit (32,000)
Deferred gross profit, Dec. 31, 2011 P128,000
15. C
16. B
2006 sales - P108,750 x 25% P27,187.50
2007 sales - P120,000 x 27.5% 33,000.00
2008 sales - P 93,750 x 28% 26,250.00
Total P86,437.50
17. B
18. D
2006 sales - P24,000 x 39% P 9,360
2007 sales - (P300,000 - P60,000 - P10,000) x 42% 96,600
2008 sales - (P480,000 - P320,000 - P5,000) x 40% 62,000
Total P167,960
19. A
Collections on 2016 installment contracts P360,000
Multiply by 33 1/3%
Realized gross profit P120,000
20. B
89
21. B
Installmet Receivable, 2015 sales, Dec. 31, 2015 P120,000
Installment Receivable, 2015 Sales (15,000)
Uncollected account on repossession (7,750)
Total P 97,250
Multiply 45%
Total 43,762.50
22. C
Cost of installment sales P263,500
Divided by Installment Sales 425,000
CGS rate 62%
GP rate (100%-62%) 38%
23. A
Installment Sales P425,000
Installment Receivable, 2016 Sales (200,000)
Collections P225,000
Multiply by GP rate 38%
Gross profit realized P 85,500
24. B
Cash sales P 90,000
Charge sales (P180,000/120%) 150,000
Installment sales (P446,400/124%) 360,000
Total sales - cash basis P 600,000
25. B
2014 2015 2016
Installment recl, beg P 74,000 P123,000
Installment sales P446,400
Installment recl, end ( 15,000) ( 45,000) ( 270,000)
Defaulted recl ( 18,000) ( 21,000)
Collections P 41,000 P 57,000 P176,400
26. A
Sales price of Article “A” P400,000
Less Overvaluation on trade -in
90
Sales price P110,000
Reconditioning cost ( 8,000)
Normal profit ( 22,000)
Market value of trade-in P 80,000
Allowed trade-in value 120,000 40,000
Adjusted sales price P360,000
Cost of Article “A” 270,000
Gross profit P 90,000
Gross profit rate 25%
27. A
Market value of trade-in P80,000
Overallowance of trade-in 40,000
Total P120,000
Multiply by GP rate 25%
Realized GP P30,000
28. A
29. C
30. B
31. B
32. C
33. C
34. C
35. A
36. D
37. A
38. C
39. C
91
40. B
Unadjusted installment sales P400,000
Add: Undervaluation of traded car (P150,000-P50,000) 100,000
Adjusted installment sales P500,000
Less: Cost of production of car 300,000
Adjusted gross profit P200,000
Divided by Adjusted installment sales 500,000
Adjusted gross profit rate based on sales 40%
Installment receivables P400,000
Down payment (25%x400,000) (100,000)
Trade in allowance (50,000)
Installment receivable balance P250,000
Annual installment (250,000/5) P 50,000
Down payment P100,000
Fair value of trade in 150,000
Annual payment- first year 50,000
Total collections in 2018 P300,000
Realized gross profit (40%x300,000) P120,000
92
REVENUE RECOGNITION
Installment Sales- Default and Repossession and Trade in merchandise
1. B
Market value of repossessed ref (P1,700 x 63%) P1,071
Unrecovered cost (P2,200 x 66%) 1,452
Loss on repossession P 381
2. A
Value assigned to repossessed merchandise:
2006 sales P 9,000
2007 sales 13,500 P22,500
Unrecovered cost
2006 sales - P22,500 x 75% P 16,875
2007 sales - P24,000 x 72.5% 17,400 34,275
Loss on repossession P11,775
3. B
Value assigned to repossessed merchandise P8,000
Unpaid balance of prior year's installment contracts defaulted P15,000
Multiply by CGS rate 60% (9,000)
Loss on repossession P(1,000)
4. C
Uncollected account of repossession P 7,750
Multiply by 55%
Total P4,262.50
Repossessions (3,000)
Loss on repossession P1,262.50
5. D
6. A
7. A
8. D
9. B
10. A
11. A
93
Estimated resale price 1,458,000
Less: Reconditioning cost 194,400
Gross profit (P1,458,000x20%) 291,600
Market Value of trade-in 972,000
Sales 6,804,000
Add Underallowance on trade-in
Trade-in allowance 960,000
Less Market Value (972,000) 12,000
Adjusted Sales 6,816,000 100%
Cost of Sales (2,726,000) 40%
Gross Profit 4,089,600 60%
Market Value of trade-in 972,000
Installment cash collection 520,000
Total Collections 1,492,000
Multiply by GPR 60%
Realized Gross Profit 895,200
12. C
Fair value of repossessed inventory P110,000
Unrecovered cost of defaulted installment receivable (P200,000x60%) 120,000
Loss on repossessions P(10,000)
13. C
Trade-in allowance P300,000
Estimated resale price P315,000
Reconditioning cost (25,000)
Gross profit (315,000x10%) (31,500) (258,500)
Overallowance on trade-in P41,500
14. A
Installment account canceled P20,000
Multiply by GP rate(50,000/250,000) 20%
Deferred gross profit written off P4,000
15. D
Installment account canceled P20,000
94
Repossessed merchandise (14,500)
Deferred gross profit written off (4,000)
Loss on repossession P1,500
16. C
Installment account canceled P40,000
Multiply by GP rate(195,000/600,000) 32.5%
Deferred gross profit written off P13,000
17. B
Installment account canceled P40,000
Repossessed merchandise (24,000)
Deferred gross profit written off (13,000)
Loss on repossession P3,000
18. C
19. D
Estimated resale price P280,000
Less: Reconditioning cost P30,000
Gross profit 70,000 (100,000)
Estimated realizable value P 180,000
20. C
Selling price P720,000
Less Overallowance (P300,000 — P180,000) (120,000)
Adjusted selling price P600,000
21. D
22. B
Uncollected installment contract P4,000
Multiply by gross margin 40%
Deferred gross profit written off P1,600
23. A
Uncollected installment contract P4,000
Repossessed merchandise (2,000)
Deferred gross profit written off (1,600)
Loss on defaults P400
24. A
95
25. B
Resale Value P13,500
Divided by 120%
Repossessed merchandise P11,250
26. C
Installment Contracts Receivable P 15,000
Multiply by 20%/120%
Deferred gross profit written off P 2,500
27. A
Installment Contracts Receivable P15,000
Repossessed merchandise (11,250)
Deferred gross profit (2,500)
Loss on repossession P1,250
28. C
29. A
Balance of Installment Contracts Receivable P 8,000
Multiply by 25%/125%
Deffered gross profir written off P 1,600
30. C
31. B
32. A
33. B
34. C
35. C
36. B
37. D
38. A
39. B
96
40. C
Inst. recl balance, Dec. 31, 2007 (P360,000 - P120,000) P240,000
Installment payment, Jan. 1 - Mar. 1 (P20,000 x 3) 60,000
Inst. recl balance, April 1 P180,000
Cost percentage x 75%
Unrecovered cost P135,000
Market value of repossessed mdse. (P13,500 - P800 - P2,700) ( 100,000)
Loss on repossession P 35,000
97
REVENUE RECOGNITION
Installment Sales- Financial Statement Presentation
1. B
Deferred gross profit before adjustment P38,000
Deferred gross profit after adjustment
2015 - P16,250 x 30/130 P 3,750
2016 - P90,000 x 33 1/3 /133 1/3 22,500 26,250
Realized gross profit P11,750
Operating expenses 1,500
Net income P10,250
2. B
Inst. Accts. Rec., end of year 3
On year 3 installment sales (P968,000 x .80 x .60) P464,640
On Year 2 installment sales (P785,000 x .80 x .25) 157,000
Total installment accounts receivable, end of Year 3 P621,640
3. A
4. B
Sales - regular P187,500
Cost of sales - regular 112,500
Gross profit - regular P 75,000
Realized gross profit (see D1) 46,875
Total gross profit P121,875
Selling expenses 31,250
Net income P 90,625
5. D
Installment Sales, 2016 P610,750
Multiply by CGS rate 60%
CGS P366,450
98
6. A
Inventory, beginning P52,500
Delivered cost of purchases 393,000
Repossessed merchandise 15,000
Cost of goods available for sale P460,500
Less Inventory, end 70,500
Cost of goods sold P390,000
Multiply by 360/600
Cost of installment sales P234,000
7. C
8. C
9. B
10. A
11. A
12. C
13. A
14. C
15. A
Inventory, January 1 P 240,000
Purchases, including freight-in 1,250,000
Repossessed Merchandise 70,000
Cost of Goods Available for Sale P1,560,000
16. B
Cost of Goods Available for Sale P1,560,000
Less Inventory, December 31 260,000
Cost of Goods Sold P1,300,000
17. B
18. A
99
19. C
Amount of Sales Amount Based on Cash Sales Ratio Allocation of CGS
Cash P300,000 P 300,000 60/400(17) P195,000
Charge 600,000 500,000 100/400 (18) 325,000
Inst. 1,500,000 1,200,000 240/400(19) 780,000
P2,400,000 P2,000,000 P1,300,000
20. A
21. B
Cash Sales Charge Sales Installment Sales Total
Sales P300,000 P600,000 P1,500,000 P 2,400,000
Cost of Sales 195,000 325,000 780,000 1,300,000
Gross Profit P105,000 P275,000 P720,000 P 1,100,000
Less Deferred Gross Profit, 2016 sales 460,800 460,800
Realized GP on 2016 sales P105,000 P275,000 P263,200 P 639,200
Add Realized GP on 2014 and 2015 sales 169,500
Total Realized Gross Profit P 808,700
Les: Loss on Repossession
51,000 Realized Gross Profit after loss on repossession
P 757,700 Operating Expenses, including bad debts
465,000 Net Income before Income Tax
P 292,700
Income Tax (292,700x35%) (20) 102,445
Net Income (21) P 190,255
22
22. D 32. A
23. A 33. C
24. B 34. A
25. A 35. D
26. A 36. B
27. B 37. B
28. D 38. D
29. A 39. A
30. C 40. C
31. B
100
REVENUE RECOGNITION
Installment Sales- PAS 18/ PFRS 15
1. B 21. B
2. A 22. B
3. C 23. D
4. D 24. C
5. D 25. C
6. C 26. D
7. A 27. A
8. C 28. D
9. B 29. C
10. A 30. C
11. A 31. A
12. C 32. B
13. A 33. C
14. C 34. B
15. B 35. A
16. B 36. D
17. D 37. B
18. A 38. C
19. A 39. C
20. A 40. D
101
REVENUE RECOGNITION
Long-term Construction Contracts- Percentage of completion
1. C
2. B
3. D
4. C
5. B
6. D
Contract Price P20,000,000
Multipy by percentage of completion (3,000,000/15,000,000) 20%
Recogized revenue in 2015 P4,000,0000
7. A 19. D
8. B 20. B
9. B 21. B
10. B 22. B
11. A 23. B
12. C 24. B
13. B 25. A
14. C 26. C
15. D 27. B
16. D 28. C
17. C 29. A
18. D
30. C
Contract price P 15,000,000
Total estimated cost (P4,650,000+P10,850,000) (15,500,000)
Total estimated loss—to be recognized in full P 500,000
31. C
Contract price P3,000,000
Total estimated cost (1,800,000)
Total estimated gross profit P1,200,000
Percentage-of-completion (600/1,800) 331/3%
Gross profit to be recognized in 2007 P 400,000
102
32. B
Contract price P20,000,000
Cost incurred to date P 11,000,000
Estimated cost to complete 5,000,000
Total estimated cost P16,000,000
Total estimated gross profit P 4,000,000
Percentage of completion 68.75%
Contract Price P20,000,000
Multiply by percentage of completion 68.75%
Revenue from long-term construction contracts P13,750,000
33. A
34. D
Recognized revenue P13,750,000
Cost of revenue (11,000,000)
Gross profit P 2,750,000
35. A
36. D
Revenue from long-term construction contracts P13,750,000
Progress Billing on Construction Contract (10,800,000)
Inevtory-Construction in Progess, net of billings P 2,950,000
37. A
38. A
39. B
103
40. B
To date Recognized in prior year/s To be recognized this year
2014
Recognized revenue P8,000,000 - P8,000,000
Cost of revenue 8,000,000 - 8,000,000
Grossprofit - - -
========== ============ ==========
2015
Recognized revenue P24,000,000 P8,000,000 (38) P16,000,000
Cost of revenue 18,000,000 8,000,000 (39) 10,000,000
Gross profit P 6,000,000 ---------------- P 6,000,000
2016
Recognized revenue P40,000,000 P24,000,000 P16,000,000
Cost of revenue 31,000,000 18,000,000 (40) 13,000,000
Grossprofit P 9,000,000 P 6,000,000 P 3,000,000
104
REVENUE RECOGNITION
Long-term Construction Contracts- Cost Recovery Method
1. A 12. D
2. A 13. B
3. B 14. D
4. D 15. C
5. D 16. D
6. D 17. D
7. D 18. A
8. A 19. C
9. B 20. D
10. B 21. C
11. D
22. C
Contract price P1,000,000
Total costs in 2018 (360,000+840,000) (1,200,000)
Realized loss 2018 P P(200,000)
Contract price as of 2019 P1,000,000
Total costs as of 2019 (800,000+250,000) 1,050,000
Cumulative gross loss for 2019 P (50,000)
Realized gross loss for 2018 (200,000)
Realized gross profit for 2019 P 150,000
23. A
Progress billings as of December 31, 2020
(1,000,000)x(30%+20%+40%) P900,000
Construction in progress as of December 31, 2020 ( 870,000)
Excess of progress billings over construction in progress
on December 31, 2020 P 30,000
105
24. B
Cumulative billings as of December 31, 2020
(1,000,000)x(30%+20%+40%) P900,000
Mobilization fee deductible from first billing (1,000,000x5%) (50,000)
Total collections of receivables as of December 31, 2020
(120,000+450,000+180,000) (750,000)
Accounts Receivable, 12/31/2020 P100,000
25. A
26. B
27. A
Amount probable to be recovered P10,000,000
Cost incurred to date (11,000,000)
Loss P(1,000,000)
28. C
29. B
30. A
31. A
32. A
33. C
34. D
35. A
36. A
To date Recognized in prior year/s To be recognized this year
2014
Recognized revenue P17,500,000 - P17,500,000
Cost of revenue 17,500,000 - 17,500,000
Grossprofit - - (34) -
106
========== ============ ==========
2015
Recognized revenue P31,500,000 P17,500,000 P14,000,000
Cost of revenue 29,250,000 17,500,000 11,750,000
Gross profit P2,250,000 ------------ (35) P2,250,000
2016
Recognized revenue P35,000,000 P31,500,000 P3,500,000
Cost of revenue 31,000,000 29,250,000 1,750,000
Grossprofit P4,000,000 P2,250,000 (36) P1,750,000
37. C
38. D
39. D
107
REVENUE RECOGNITION
Long term construction contract – financial statement presentation
1. B 20.D
2. C 21.C
3. A 22.B
4. B 23.A
5. A 24.D
6. D 25.C
7. C 26.C
8. B 27.B
9. B 28.C
10.D 29.A
11.D 30.D
12.C 31.B
13.D 32.B
14.C 33.A
15.D 34.B
16.B 35.C
17.C 36.C
18.D 37.A
19.C
38.B
39.C
Costs incurred to date as of December 31, 2019 (440,000+680,000) 1,120,000
108
Contract price 1,500,000
Multiply by ____80%
40. A
109
REVENUE RECOGNITION
Computation of gross amount due from to customer
1. C.
Percentage of completion method:
Total (1,200,000)
2. B.
3. A.
Contract Price 210M
Less: Total estimated cost 180M
Estimated profit 30M
Multiply by 90/180
Contract Price earned 15M
4. D.
P1 P2
CP 2,100,000 750,000
TC 900,000 875,000
X 66.67%
110
5. C.
Contract Price P100,000,000
Multiplied by: Gross profit rate 25%
Estimated Gross profit of the entire contract P 25,000,000
Multiplied by: Percentage of completion for first year 50%
Gross profit realized for current year P 12,500,000
6. D.
7. B.
8. B.
9. C.
10.B.
11.C.
12.D.
13.A.
14.C.
15.D.
Mobilization fee (200,000,000 x 5%) 10,000,000
16.D.
Cost of materials, labor and overhead 635,200
Materials set aside (specific project) 50,000
Incidental income from surplus materials ( 10,000)
Cost incurred to date 675,200
17.A.
Cost incurred to date 675,200
Total estimated cost (750,000 + 50,000) ÷ 800,000
Percentage of completion 84.40%
111
20.B.
21.D.
22.C.
23.D.
24.C.
25.D.
26.D.
Contract price 10,500,000
Total estimated cost 7,350,000
Estimated gross profit 3,150,000
Multiplied by 36%
Realized gross profit 1,134,000
27.A.
Contract price 2,100,000
Multiplied by 21%
28.C.
CP 2,100,000
% of C 64.25%
CIP 1,349,250
CITD (1,394,250)
GPTD ( 45,000)
GPPYs ( 15,750)
GP-2021 ( 60,750)
29.C.
PB 525,000
Adj. PB 1,680,000
PB-2021 ( 330,750)
CIP 1,349,250
30.A.
31.C.
112
Contract price 1,000,000
Less: Total costs as of 2019 (800,000+250,000) (1,050,000)
Cumulative gross loss for 2019 ( 50,000)
Realized gross loss for 2018 ( 200,000)
Realized gross profit for 2019 150,000
32.A.
Progress billings as of 12/31/2020 (1M) x (30%+20%+40%) 900,000
Less: Construction in progress as of 12/31/2020 870,000
33.B.
Cumulative billings as of 12/31/2020 (1M) x (30%+20%+40%) 900,000
34.A.
35.B.
36.C.
37.B.
38. D.
20,000,000 x (3,000,000/15,000,000) = 4,000,000
113
39.A.
Contract price 10,500,000
Less: total estimated cost
Cost incurred to date 3,150,000
Est. cost to complete 6,300,000 9,450,000
Total estimated income 1,050,000
% of completion (3150/9450) 33.33%
Income to be recognized in 2015 350,000
40. B.
Contract price 9,000,000
Total estimated cost 8,100,000
Total estimated income 900,000
% of completion (27/81) 33.33%
Income to be recognized last year 300,000
114
REVENUE RECOGNTION
1. D
2. A
List price P1,000
115
However, total contract costs are expected to exceed total expected contract
revenue, therefore an additional loss must be recognized in respect of the
onerous contract, i.e. 18.18% (the percentage of future activity on the
contract) × P100,000 the excess of total expected contract costs over total
expected contract revenue = P18,182. Expense = P908,182 (i.e. P890,000
incurred to date + P18,182 in respect of the onerous contract).
9.) A
2011:
Costs incurred relating to work performed to date P200,000
Divided by total expected contract costs 600,000
Stage of completion 33.33%
2012:
Costs incurred relating to work performed to date P600,000
Divided by total expected contract cost 750,000
Stage of completion 80%
2013:
Contract price P1,000,000
Less (contract revenue recognized up to the end of 2012) 800,000
Contract revenue recognized in 2013 P200,000
10. B
2011:
Costs incurred relating to work performed to date P200,000
Divided by total expected contract costs 600,000
Stage of completion 33.33%
116
Multiply by contract price P1,000,000
Contract revenue recognized in 2011 P333,333
2012:
2013:
Contract price P1,000,000
Less (contract revenue recognized up to the end of 20X2) 733,333
Contract revenue recognized in 20X3 P266,667
11. C
12. C
13. A
14. D
15. B
16. D
17. C
Costs that cannot be attributed to contract activity or cannot be allocated to a
contract are excluded from the costs of a construction contract. Such costs
include:
(a) general administration costs for which reimbursement is not specified in
the contract;
(b) selling costs;
117
(c) research and development costs for which reimbursement is not specified
in the contract; and
(d) depreciation of idle plant and equipment that is not used on a particular
contract.
18. D
When a contract covers a number of assets, the construction of each asset
shall be treated as a separate construction contract when:
(a) separate proposals have been submitted for each asset;
(b) each asset has been subject to separate negotiation, and the contractor
and customer are able to accept or reject that part of the contract relating to
each asset; and
(c) the costs and revenues of each asset can be identified.
19. A
A group of contracts, whether with a single customer or with several
customers, shall be treated as a single construction contract when:
(a) the group of contracts is negotiated as a single package;
(b) the contracts are so closely interrelated that they are, in effect, part of a
single project with an overall profit margin; and
(c) the contracts are performed concurrently or in a continuous sequence.
20. C
21. C
For each construction contract in progress at the reporting date the gross
amount due from customers is shown as an asset in the statement of
financial position at the net amount of:
(a) total costs incurred on the contract plus the cumulative recognized profit
(or less the cumulative recognized loss); less
(b) progress billings (i.e. amounts actually invoiced to customers for work
performed on a contract whether or not they have been paid by the
customer).
118
22. C 32. A
23. A 33. C
24. B 34. B
25. B 35. C
26. B 36. D
27. D 37. C
28. D 38. D
29. B 39. C
30. B 40. B
31. B
119
REVENUE RECOGNTION
Franchise Operations – Initial Franchise Fee
1. A
2. A
3. A
4. C
5. D
6. C
7. C
8. A Deferred revenue: 500,000 * 3.60478 = 1,802,390
9. C
10.B
11.C
12.D Continuing fee (P400,000 x 5%) P20,000
13.A Initial franchise fee P50,000
Add Percentage on revenue (P400,000 x 5%) P20,000
Total revenue P70,000
14.B
15.B
16.C
17.B
18.B Continuing fee (P400,000 x 10%) P40,000
19.D Initial franchise fee P500,000
Add Percentage on revenue (P400,000 x 10%) P 40,000
Total revenue P540,000
20.B Allocated revenue to construction of stall (P400,000 x 20/50) P160,000
120
Initial franchise fee revenue P440,183
24.D Cash down payment P200,000
Present value of note receivable 240,183
Initial franchise fee revenue P440,183
Less: Direct cost of initial franchise fee 352,146
Gross profit under accrual basis P 88,037
25.A Interest Income for year 2018 (P240,183 x 12%) P28,822
26.A Contingent franchise fee revenue (P50,000 x 8%) P4,000
27.D Gross profit under accrual basis P88,037
Add: Interest Income for year 2018 (P240,183 x 12%) 28,822
Add: Contingent franchise fee revenue (P50,000 x 8%) 4,000
Less: Indirect cost – Expense as incurred (22,009)
Net income under accrual basis P98,850
28.B
29.C Cash P 400,000
Add payments (P200,000 x 3.6299) 725,980
Total revenue P1,125,980
30.B Cash P 400,000
Add payments (P200,000 x 3.6299) 725,980
Total revenue 1,125,980
Less cost 400,000
Gross profit P 725,980
Add interest (P725,980 x .04) 29,039
Total P 755,019
31.C Down payment P300,000
Multiply by 40%
Total P120,000
32.D
33.B Down payment P300,000
Add payments (P100,000 x 1.336) 133,550
Total revenue P473,550
34.C Down payment P300,000
Add payments (P100,000 x 1.336) 133,550
Total revenue P473,550
Less cost (P5,000,000 x 5%) 250,000
Total P723,550
35.D 38.C
36.A 39.A
37.B 40.C
121
REVENUE RECOGNITION
Franchise operations- Continuing franchise fee, bargain purchase
option, commingled revenue
1. A
3. A
4. B
5. B
7. A P102,000
8. D P120,000
9. A
122
REVENUE RECOGNTION
Franchise Operations – Repossessed Franchise
1. A 9. D
2. D 10.A
3. C 11.B
4. A 12.B
5. C 13.A
6. D 14.B
7. D 15.C
8. A 16.A
17.B
Unrecovered cost:
Unpaid balance 15,000 16,000
Less: Deferred gross profit
2018: 15,000 x 25% 3,750
2019: 16,000 x 27.5% _______ 4,400
Unrecovered cost 11,250 11,600
Value of repossess merchandise 6,000 9,000
Loss on repossession ( 5,250) ( 2,600) (7,850)
123
19.B
124
REVENUE RECOGNTION
Franchise Operations – Option to Purchase Franchise Outlet
1. C
2. D
3. B
4. B
5. A
6. B
7. A
8. B
9. A
10.A
11.B
REVENUE RECOGNITION
Franchise Operations – Financial Statement Presentation
1. C 21.D
2. C 22.B
3. A 23.A
4. C 24.D
5. B 25.B
6. A 26.A
7. C 27.C
8. D 28.A
9. B 29.B
10.A 30.C
11.A 31.A
12.D 32.A
13.A 33.A
14.B 34.C
15.C 35.C
16.A 36.C
17.A 37.D
18.A 38.C
19.A 39.A
20.D 40.A
125
126
REVENUE RECOGNTION
Franchise Operations – Accounting for SMEs
20.D
1. A 21.B
2. A 22.C
3. C 23.A
4. A 24.A
5. D 25.A
6. A 26.A
7. A 27.A
8. A 28.D
9. A 29.A
10.B 30.D
11.B 31.A
12.C 32.A
13.B 33.A
14.A 34.A
15.A 35.B
16.A 36.A
17.A 37.A
18.B 38.B
19.B 39.B
40.A
127
REVENUE RECOGNITION
Consignment Sales
1. C
2. B
3. B
4. C
5. A
6. C Under a consignment sales arrangement, the consignor ships merchandise
to the consignee who acts as agent for the consignor in selling the goods. The
goods are in the physical possession of the consignee but remain the property of
the consignor and are included in the consignor’s inventory count. Sales revenue
and the related cost of goods sold from these consigned goods should be
recognized by the consignor only when the merchandise is sold and delivered to
the ultimate borrower. Accordingly, recognition occurs when notification is
received that the consignee has sold the goods.
Answer (A) is incorrect because, at the date of shipment, the goods are still the
property of the consignor. Answers (B) and (D) are incorrect because the
consignee does not recognize sales revenue or cost of goods sold for these
goods. The consignee recognizes commission revenue only when the goods are
sold and delivered to the third party.
7. A
Receipts ( 7 dozens x 12 x P2,000) P168,000
Charges: Expenses P 3,000
Commissions (15% x P168,000) 25,200 28,200
Remittance P139,800
8. A
Sales 7 x 12 x P2,000 P168,000
Cost of Sales 7 x 12 x P1,000 P84,000
Freight 7 x P30 210
Expenses 3,000
Commission P168,000 x 15% 25,200 112,410
P 55,590
9. B
128
10.A
Receipts (215 x P500) P107,500
Less Shipping charges 2,100
Remittance P105,400
11.B
(215 x 40% x P580) + (215 x 60% x P640) – (215 x P24,940
P500) =
16.A
17.D The consignee should debit consignment-in for the freight costs.
Consignment-in is a receivable-payable account used by consignees. It
represents the amount payable to the consignor if it has a credit balance. If it
has a debit balance, it reflects the amount receivable from the consignor. Before
consigned goods are sold, expenditures chargeable to the consignor are recorded
in the consignment-in account as receivable. After the consigned goods are sold,
the consignee’s net liability to the consignor is reflected in the account.
Answers (A), (B), and (C) are incorrect because the freight costs constitute a
receivable.
18.B Answer (B) is correct. ABC debits the cash received $43,000 [$50,000
sales - $2,000 advertising - (.10 x $50,000) sales commission]. The advertising
129
and commission expenses are debited for $2,000 and $5,000, respectively.
Finally, $50,000 of gross revenue is credited.
Answer (A) is incorrect because the freight was paid earlier in the period and
would have been recorded then by a credit to cash and a debit to inventory.
Thus, the freight costs will be released to income via cost of goods sold. Answer
(C) is incorrect because the 10% commission and the advertising costs are
ignored in this answer. Answer (D) is incorrect because the reimbursable
advertising costs are ignored in this answer.
19.A
20.A
21.D
22.C
23.A
24.C
25.D
26.D
27.C
Remittance P64,980
Charges:
Delivery expense P 850
Repairs 2,000 2,850
Total P67,380
÷ 85%
Sales P79,800
28.D
Remittance P64,980
Charges:
Delivery expense P 850
Repairs 2,000 2,850
Total P67,380
÷ 85%
Sales P79,800
Cost of sales 52,000*
Gross profit P27,800
Expenses:
Commission (P79,800 x 15%) P11,970
Repairs (P2,000 x 60/100) 1,200
130
Delivery 850
Shipping cost (P900 x 260/300) 780 14,800
Consignment profit P13,000
*Sales P79,800
Less Sales of units with defects (200 x 60,000
P300)
Sales of repaired units P19,800
Selling price of repaired units ÷ P330
Number of repaired units that were sold 60
Units sold without repairs 240
Total number of units sold 300
Unit cost x P200
Cost of sales
P52,000
29. B
Sales P79,800
Less Sales of units with defects (200 x 60,000
P300)
Sales of repaired units P19,800
Selling price of repaired units ÷ P330
Number of repaired units that were sold 60
30. C
Remittance P68,250
Consignee charges, excluding the 15% commission
(P4,500 + P3,000 + P750) 8,250
Sum of remittances and charges P76,500
÷ 85%
Sales price of 6 refrigerators P90,000
131
32. A
Sales P90,000
Cost of sales (6 x P9000) 54,000
Gross profit P36,000
Expenses:
Commission (P90,000 x 15%) P13,500
Freight-out (P6,000 x 6/10) 3,600
Marketing expense 4,500
Delivery and installation 3,000
Cartage (P750 x 6/10) 450 25,050
Net profit from the sale of consigned P10,950
goods
33. D P90,000 x 15% = P13,500
34. C
35. B
36. C
37. D
38. D
39. B
40. C
132
ACCOUNTING FOR HOME OFFICE, BRANCH AND AGENCY
TRANSACTIONS
Transactions on the books of the Home Office and the Branch
1. A
2. C
3. B
7. A P50,400/120% = P42,000
10. E
11. A
12. A
13. B
133
14. B In a periodic system, when merchandise is received by a branch from the home
office, the merchandise should be reflected as a shipment from the home office in the
amount of the transfer price, with a corresponding entry to the home office account
to indicate the equity of the home office in the net assets of the branch.
17.
18. B
19. B
21. C
Home Office Books:
Davoo Branch 39,000
STB, cost 32,500
Unrealized Profit 5,200
Cash (freight) 1,300
134
BC - Boguio 19,630
Excess freight 520
BC-Davao 20,150
Davao Branch:
SFHO 37,700
Freight-in 1,300
HOC 39,000
HOC 20,150
SFHO (50%) 18,850
Freight-in 650
Cash (freight) 650
Baguio Branch:
SFHO 18,850
Freight-in 780
HOC 19,630
23. A
24. D
26. A
135
28. B Allowance before adjustment (TGAS from HO) P48,000
Less: allowance on the branch ending inventory (from HO)
[(P40,000-16,000)/120%] x 20% 4,000
Realized allowance (Overstatement of cost of sales in the branch)P44,000
*Since there are no shipments in transit and there was no error in recording shipments,
therefore, the shipments from office account was correctly recorded, so, to compute for
the billing price would be: 187,500/150,000 = 25%. Markup on cost would be 25%
31. A
32. B In branch accounting, the branch office account on the books of the home
office represents the investment by the home office in the net assets of the
branch, not the branch’s equity in the home office.
33. C When goods are shipped from a home office to a branch at a transfer price
that reflects original cost plus a markup, the branch must record the
shipment at the transfer price. The home office most often reflects the
shipments to branch at original cost. To maintain a reciprocal relationship
between the home office and the branch office accounts, an unrealized profit
in branch inventories account reflects the markup.
136
34. C
37. B
38. C
137
ACCOUNTING FOR HOME OFFICE, BRANCH AND AGENCY
TRANSACTIONS
Reconciliation of reciprocal accounts
2. D P179,920
3.
5.
7. A
138
Branch Books Home Office Current (Cr.)
Unadjusted P506,700
Settlement check (40,000)
Merchandise return (15,000)
General expenses 28,000
Insurance premiums (P600 was recorded) 900
Adjusted Balance P535,000
9. D
10. D
Inv inBranch Home Office
Unadjusted balance P102,000 P52,800
Branch remittance not recorded by home office (12,000)
Shipments not recorded by the branch 24,000
Unrecorded branch expenses 6,000
Customer’s remittance to home not recorded by
the branch (3,600)
Erroneous recording of branch shipments
(51,600-40,800) 10,800
Adjusted balances P90,000 P90,000
12. D
13. A
(Branch Books) (Home Office Books)
Home Office Inv. in Nova Branch
Unadjusted balances P27,350 P25,550
Error in recording shipment to QC branch (12,000)
Error in recording shipment to Nova branch 15,000
Branch AR collected by home office (3,600)
Merchandise returns in transit ( 1,200)
Error in recording branch profit ( 3,600)
Adjusted balances P23,750 P23,750
139
14. C True Branch Net Income P156,000
Less: branch net income as reported by the branch 60,000
Overvaluation of CGS 96,000
Less: Cost of good sold from the home office at BP
Inventory, December 1 P70,000
Shipments from HO 350,000
COGAS P420,000
Less: Inventory, December 31 84,000 336,000
CGS from the home office, at cost P240,000
140
19. C
21. C
22. B
Marketing Expense of another branch charged to Butuan P (10,000)
Butuan’s remittance credited to Davao ( 65,700)
Net adjustment in Home Office Banch account P (75,700)
23. B
Fixed account not recorded by Butuan P (53,960)
Inventory transfer recorded twice by Butuan 75,000
Error in recording debit memo 4,650 (90)
Net adjustment in Branch Books P 20,950
141
25. A
(Branch Books) (Home Office Books)
Home Office Investment in Branch
Unadjusted balances P 440,000 P 496,000
Branch AR collected by Home Office ( 8,000)
Shipments in transit 32,000
Acquisition of furniture (12,000)
Merchandise returns (15,000)
Cash remittance in transit - ( 5,000)
Adjusted balances P 464,000 P 464,000
27. C
28. A
Unadjusted balance – Investment in Branch account, 12/31 P430,000
Charge for advances by president (5,500)
Erroneous entry for merchandise allowance ( 600)
Share in advertising expense (9,000)Unadjusted
balance – Home Office account, 12/31 P414,900
30. B
142
31. B
35. D
143
37. A Branch Acct. HO Acct.
Balances before adjustment P150,000 P117,420
Adjustments:
1. Shipments in transit 37,500
2. HO AR collected by branch 10,500
3. Supplies returned (4,500)
4. Error in recording Br. net income (1,080)
5. Cash to Branch in transit 25,000 25,000
Total P179,920 P179,920
38. D P179,920
39. C
Home Office Books Branch Books
(Branch Current- Dr. bal.) (HO Current - Cr. bal.)
Unadjusted balance P150,000 P117,420
Add: (deduct) Adjustments:
In Transit - 37,500
HO A/R collected by br. 10,500
Supplies returned (4,500)
Error in recording Br. NI (1,080)
Cash sent to branch to
General Exp by HO 25,000 25,000
Adjusted Balance P179,920 P179,920
144
ACCOUNTING FOR HOME OFFICE, BRANCH AND AGENCY
TRANSACTIONS
Preparation of Individual and Combined Financial Statements
4.
5. B
6. A
7. A
8. A Sales P112,500
Cost of Sales:
Shipments from home office P120,000
Less Inventory, Dec. 31 30,000 90,000
Gross profit P 22,500
Expenses 8,100
Net Profit P 14,400
145
11. B
Assets:
Inventory, December 31 P19,000 P12,000
Imprest branch fund 2,000 1,500
Accounts Receivable, December 31 70,000 53,500
Total Assets P91,000 P67,000
Less: Liabilities 0 0
Home Office Current Account P91,000 P67,000
*Deferred profit on the branch beginning inventory from the home office
146
18. B Sales P 100,000
Cost of sales
Inventory, beg. P21,000
Merchandise from Home Office 61,000
Merchandise available for sale P82,000
Less Inventory, end 19,000 63,000
Gross profit P37,000
Operating Expenses 21,000
Net profit of Branch A P16,000
20. A
22.
24. D
25. D
Sales (P100,000-P33,000+P50,000) P 117,000
Less: Cost of goods sold:.
lnventory, beg. [P 15,000 + (P5,500 /110%)
or (P5,500 - P500)] P20,000
Add: Purchases (P50,000 + P7,000) 57,000
147
COGAS P77,000
Less: lnventory, end
[P11,000 + P1,050 + (P6,000- P1,050]/110% P16,550 60,450
Gross profit P 56,550
Less: Expenses {P20,000 + P6,000 + P5,000) 31,000
Cornbined Net income P25,550
26.
27.
28.
29.
36. B
Assets:
Inventory, December 31 P19,000 P12,000
Imprest branch fund 2,000 1,500
Accounts Receivable, December 31 70,000 53,500
Total Assets P91,000 P67,000
Less: Liabilities 0 0
Home Office Current Account P91,000 P67,000
148
37. A Sales P112,500
Cost of Sales:
Shipments from home office P120,000
Less Inventory, Dec. 31 30,000 90,000
Gross profit P 22,500
Expenses 8,100
Net Profit P 14,400
149
ACCOUNTING FOR HOME OFFICE, BRANCH AND AGENCY
TRANSACTIONS
Special procedures in home office and branch transactions
(inter-branch transfer of cash and merchandise at cost or at billed price)
1. D
2. D
3. B
4. C
6. A
True Branch net income P156,000
Less: Branch net income as reported (by the branch) 60,000
Over valuation of Cost of goods sold P96,000
Less: Cost of goods sold from home office at billed price:
Inventory, December 1 P70,000
Shipment from home office 350,000
COG from home office available for sale P420,000
Less: Inventory, December 31 84,000 336,000
Cost of goods sold from home office, at cost P240,000
7. B
Allowance for overvaluation after adjustment:
P84,000 x 40/140 = P24,000
150
8. B
9. B
10. C
11. D
12. D
14. B
15. D
16. D
20. A
151
21. C
22. C
BC - Boguio 19,630
Excess freight 520
BC-Davao 20,150
Bukidnon Branch:
SFHO 37,700
Freight-in 1,300
HOC 39,000
HOC 20,150
SFHO (50%) 18,850
Freight-in 650
Cash (freight) 650
Baguio Branch:
SFHO 18,850
152
Freight-in 780
HOC 19,630
28. C
35. A
153
36. C Imprest branch fund P 1,500
Accounts Receivable, Dec. 31 53,000
Inventory, Dec. 31 12,000
Balance of Branch account - current P 67,000
37.
154
39. C
40. A
155
ACCOUNTING FOR HOME OFFICE, BRANCH AND AGENCY
TRANSACTIONS
Accounting for Agency Transactions
1. B
2. A
3. B Sales P400,000
Cost of sales ( 400,0000 - 70,000) 330,000
Gross profit 70,000
Expenses [30,000 + 10,000 +
(10,000 - 6,000) + 5,000] 49,000
Net profit P 21,000
4. B Sales P46,500
Multiply 70%
Cost of sales w/o freight P32,550
Add freight 1,100
Cost of sales w/ freight P33,650
5. B Sales P46,500
Less Sales Discount (39,690 / 98%) - 39,690 810 P45,690
Cost of sales 33,650
Gross Profit P12,040
Expenses:
Selling P 2,820
Administrative (46,500 x 5%) 2,325
Samples Expenses 1,900 7,045
Net Profit P 4,995
7. C Sales P176,000
Cost of sales 105,000
Gross Profit P 71,000
Expenses 39,750
Net Income P 31,250
9. B Sales P 87,500
Cost of sales 70,000
156
Gross Profit P 17,500
Expenses (350 + 250) 6,000
Net Income P 11,500
14. A In adopting the imprest system for the agency working fund, the home office
writes a check to the agency for the amount of the fund. Establishment of the
fund is recorded on the home office books by a debit to the Agency working fund
and credit cash. The
agency will request fund replenishment whenever the fund runs low and at the
end of each fiscal period. Such a request is normally accomplished by an
itemized and authenticated statement of disburserments and the paid vouchers.
15. B
157
Misc 2,000
Salaries 15,000
Commission 11,000
Samples 7,500
Advertising 1,000
Net income P22,400
17. C
19. A In adopting the imprest system for the agency working fund, the home office
writes a check to the agency for the amount of the fund. Establishment of the fund is
recorded on the home office books by a debit to the Agency working fund and credit
cash. The agency will request fund replenishment whenever the fund runs low and at
the end of each fiscal period. Such a request is normally accomplished by an itemized
and authenticated statement of disbursements and the paid vouchers. Upon sending
the agency a check in replenishment of the fund, the home office debits expense.
20. B
21. B
22. C
23. D
24. D
158
26. B Allowance before adjustment (TGAS from HO) P48,000
Less: allowance on the branch ending inventory (from HO)
[(P40,000-16,000)/120%] x 20% 4,000
Realized allowance (Overstatement of cost of sales in the branch)P44,000
28. C
31. B
159
(10,000 - 6,000) + 5,000] 49,000
Net profit P 21,000
40. A In adopting the imprest system for the agency working fund, the home office
writes a check to the agency for the amount of the fund. Establishment of the
fund is recorded on the home office books by a debit to the Agency working fund
and credit cash. The agency will request fund replenishment whenever the fund
runs low and at the end of each fiscal period. Such a request is normally
accomplished by an itemized and authenticated statement of disbursements and
the paid vouchers.
160
BUSINESS COMBINATION
THEORIES
1. B 10. A
2. B 11. A
3. D 12. A
4. A 13. A
5. C 14. A
6. D 15. A
7. C 16. A
8. D 17. A
9. A 18. A
PROBLEMS
19. D
20. D
161
21. D
Goodwill P284,000
Adjustments Contingent Consideration (14,000)
Goodwill 270,000
22. A
23. A
Cash P400
Shares 360
Total P 760
Goodwill P55
24. A
Receivables:
Acquirer P480
Acquiree 160
Total P 660
25. A
Inventory:
Acquirer P660
Acquiree 300
Total P 960
162
26. D
Buildings:
Acquiree 280
Total P 1,480
27. B
Long-term Liabilities:
Acquirer P1,140
Acquiree 300
Total P 1,440
28. C
Common Stock:
Acquirer P1,200
Acquiree 0
Total P 1,400
29. A
Retained Ernings:
Acquirer P1,080
Acquiree 0
Total P 1,065
163
30. B
APIC:
Acquirer P-0-
Total P 150
31. D
Cash:
Acquirer P900
Acquiree 80
Total P 555
32. B
Total P2,200,000
33. A
164
34. B
Consideration P160,000
Income (20,000)
35. A
Consideration P160,000
Income (20,000)
36. A
Consideration P800,000
Gain P500,000
37. A
38. A
39. A
40. C
Consideration P600,000
Goodwill P112,500
165
BUSINESS COMBINATION
Recognition of Acquired Assets and Liabilities
THEORIES
1 B 11 A
2 D 12 A
3 B 13 A
4 D 14 A
5 B 15 A
6 C 16 A
7 A 17 A
8 B 18 D
9 A 19 D
10 A 20 D
PROBLEMS
21. C
22. C
23. A
24. C
BV (140,000)
Loss P30,000
166
25. D
26. D
27. B
(50,000+8,000+100,000)
28. C
Salaries 16,000
29. B
Total P 19,000
30. C
31. C
32. B
33. B
167
34. A
35. D
36. C
37. C
38. A
39. A
40. A
168
BUSINESS COMBINATION
Recognition and Measurement of Goodwill and Gain from a
Bargain purchase
THEORIES
1 B 13 A
2 B 14 A
3 C 15 C
4 C 16 C
5 A 17 C
6 C 18 B
7 C 19 B
8 C 20 B
9 B 21 C
10 A 22 C
11 C 23 C
12 C
PROBEMS
24. A
Proceeds P620,000
169
25. C
Proceeds P1,800,000
26. D
27. B
Proceeds P1,240,000
28. B
Proceeds P 250,000
Fair value- net assets (180,000)
Goodwill P 70,000
29. B
Goodwill 5,000
170
30. C
BV (140,000)
Loss P30,000
31. B
BV (28,000,000)
Loss P4,000,000
32. A
Consideration(37,000*P40) P1,500,000
Gain P100,000
33. D
Gain P200,000
34. C
35. B
171
36. B
Considerations P600,000
Goodwill P180,000
37. C
Total 143,000
38. C
Total 51,000
39. A
40. D
Total P1,800,000
172
BUSINESS COMBINATION
Journal entries
1 A 21 A
2 B 22 C
3 A 23 B
4 A 24 D
5 B 25 B
6 A 26 A
7 E 27 C
8 D 28 C
9 D 29 A
10 D 30 D
11 D 31 B
12 C 32 D
13 A 33 C
14 C 34 C
15 C 35 A
16 D 36 D
17 B 37 D
18 D 38 D
19 A 39 A
20 A 40 A
173
BUSINESS COMBINATION
FS Presentation
1. C 23.A
2. B 24.D
3. D 25.D
4. C 26.C
5. B 27.D
6. D 28.B
7. B 29.B
8. A 30.A
9. A 31.C
10.D 32.E
11.D 33.B
12.A 34.A
13.C 35.B
14.A 36.A
15.C 37.B
16.C 38.B
17.D 39.D
18.D 40.C
19.D
20.D
21.A
22.A
174
BUSINESS COMBINATION
1. C 6. A 11. B
2. D 7. D 12. C
3. B 8. B 13. A
4. C 9. D 14. B
5. B 10. D 15. B
16. D
17. B.
Goodwill P190,000
18. C 23. C
19. A 24. A
20. B 25. A
21. B 26. C
22. A
175
27. D
Advisory 1,250
Legal 500
Accounting 150
Valuation 100
28. D
Advisory 1,250
Legal 500
Accounting 150
Valuation 100
176
29. A
Present value of the P30,000 deferred patent discounted by 5% per annum for 2 years.
no. Of years discounted
= future value / [(1 + discount rate per annum)
= 30,000/ [(1.05)2]
= P27,211
30. C
Valuation Input
Level
Total 78,000
31. A
32. D
Patent 2,000
Land 2,500
Equipment 550
Inventory 400
Payables 180
177
Provisions (short-term) 220
Goodwill 516
33. C
178
Total Deferred Tax (568)
Liability
34. B
Goodwill P 516
35. A
36. D
Fair Value
Land 2,000
Inventory 280
179
37. A
Patent 500
38. A
Goodwill P 698
39. D
180
Cost of business combination P 3,308
Goodwill P 1,020
40. A
181
term)
Total (322)
deferred
tax
liability
182
1. D 22. D
2. A 23. D
3. B 24. A
4. C 25. B
5. A 26. A
6. C 27. D
7. C 28. A
8. D 29. D
9. D 30. B
10. C 31. C
11. B 32. C
12. C 33. B
13. B 34. C
14. B 35. C
15. C 36. B
16. D 37. A
17. C 38. D
18. B 39. B
19. A 40. C
20. A
21. D
183
SEPARATE FINANCIAL STATEMENTS
1. A
2. C
3. D
4. C
5. C
6. A
Goodwill P240,000
7. A
Goodwill 240,000
184
8. B
9. B
Cash P 100,000
Inventory 250,000
Land 180,000
Building 300,000
10. D
11. B
12. A
13. C
14. A
15. A
16. D
185
17. C
PARENT SONS
18. C
19. C
PHILIPS SIPS
20. C
21. C
22. B
23. B
24. C
25. A
186
Income Statement
Statement of Retained
Earnings
Balance Sheet
26. A
27. D
28. A
187
29. A
30. A
31. C
Pimsol Shipping
Corp.
Income Statement
Statement of Retained
Earnings
Balance Sheet
188
32. A
less: Elaine's retained earnings, end of the current reporting period 1,400,000
900,000
33. C
34. C
35. C
36. A
189
37. B
38. D
90,000
200,000 + 100,000
39. B
Adjustments 35,000
40. A
The building and Equipment's book value was overvalued relative to the fair value.
190
SEPARATE FINANCIAL STATEMENTS
Accounting for SME
1. D 21.A
2. C 22.A
3. C 23.A
4. D 24.D
5. C 25.D
6. D 26.A
7. D 27.C
8. C 28.C
9. C 29.D
10.B 30.B
11.B 31.C
12.D 32.B
13.D 33.C
14.D 34.B
15.D 35.A
16.D 36.A
17.A 37.C
18.C 38.D
19.A 39.B
20.C 40.A
191
CONSOLIDATED FINANCIAL STATEMENTS
Date of Acquisition
1. B 16.B
2. C 17.C
3. A 18.D
4. C 19.C
5. D 20.B.
6. C
21.B
7. B
22.C
8. B
23.C
9. A 24.C
10.A 25.D
11.C 26.B
12.B 27.D
13.C 28.C
14.B 29.A
15.A 30.C
31.D
32. C. P240,000
Cost of investment P 232,000
Less: Goodwill from business combination ( 40,000)
Book value of investment P 192,000
÷ 80%
Underlying book value of T Deck’s net assets P 240,000
33. B. P202,000
Riger net income from operations P 200,000
Dividend revenue (10% x P20,000) 2,000
Net income P 202,000
34. B. P232,000
35. B. P264,000
192
Controlling income Ruger + Nina P 280,000
Noncontrolling interest (20% x P80,000) ( 16,000)
Controlling interest P 264,000
36. A. P580,000
P500, 000 book value + 80,000 = P580,000
37. A. P576,000
P600,000 book value - 24,000 = P576,000
38. A. P24,000
P120,000 book value – 96,000 = P24,000
39. A. P60,000
40. A. P250,000
193
Subsequent to Date of Acquisition
1. A 13.C
2. C 14.A
3. D 15.C
4. B 16.B
5. A 17.B
6. C 18.C
7. C 19.C
8. D 20.D
9. C 21.D
10.A 22.C
11.B 23.C
12.D
24. C. P75,000
Cost of Investment (100%) P310,000
Fair Value of identifiable assets
Cash P 30,000
Merchandise Inventory 75,000
Plant and equipment 190,000
Liabilities ( 60,000) 235,000
Goodwill from business combination P 75,000
25. C. P95,000
Cost of Investment P 975,000
NCI 220,000
Total P1,195,000
FV of SNA 1,100,000
Allocated to Goodwill P 95,000
26. A. 0
Cost of Investment P 275,000
Fair value of investment (P350,000x80%) 280,000
Gain from business combination P 5,000
Therefore, there is no goodwill recognized
27. B. P454,000
194
Cost of Investment:
Common shares (24,000 x 20) P 480,000
Preferred shares (12,000 x 100) 1,200,000
Cash 240,000 P 1,920,000
Fair value of identifiable net assets acquired:
Accounts receivable P 158,000
Inventory 412,000
Land 540,000
Bldgs. and Equipt. 1,032,000
Current liabilities ( 228,000)
Bonds payable ( 448,000) P 1,466,000
Goodwill from business combination P 454,000
28. B. P1,800,000
Fair value of identifiable net assets acquired:
Current assets P 400,000
Property and equipment 1,600,000
Liabilities ( 400,000) P 1,600,000
Add: Goodwill from business combination 200,000
Cost of Investment P 1,800,000
29. A. P60,000
Average earnings (50,000 + 60,000/2) P 55,000
Normal Earnings 25,000
Excess earnings P 30,000
x 2
Goodwill still be recognized P 60,000
30. B. P400,000
Cost of Investment P 1,700,000
Fair Value of Net Assets 1,300,000
Goodwill P 400,000
31. C. P100,000
Minority interest (MINA) P 35,100
÷ 30%
Stockholders’ equity of subsidiary, Nadir P 117,000
× 70%
Book value of investment P 81,900
Add: Allocated excess to fixed assets 10,000
Allocated excess to goodwill 8,100
Cost of investment P 100,000
32. D. P81,900
195
Book value of investment or underlying equity in Nadir net assets:
Stockholders’ equity of subsidiary, Nadir (35,100/30%) P 117,000
× 70%
P 81,900
33. B. P25,000
34. B. P500,000
Cost of Investment P 2,500,000
Stockholders’ Equity 2,000,000
Goodwill P 500,000
35. C.
FMV of shares issued by Pyramid (20,000 x P28) P 560,000
Finder’s fees 10,000
Legal and accounting fees 6,000
Total cost of investment P 576,000
36. A. P146,000
Investment cost P 576,000
FV of Sphinx’s net assets (680,000+50,000-300,000) 430,000
Goodwill P 146,000
37. A. P180,000
Normal earnings for similar firms = P1,750,000 × 14% = P245,000
Excess earnings = P290,000 – 245,000 = P45,000
Goodwill = P45,000 × 4 years = P180,000
38. A. P131,117
Goodwill = P45,000 × 2.91371 = P131,117
39. A. P250,000
Goodwill = [P1,300,000 – (P1,750,000 – 700,000)] = P250,000
40. B. P4,860,000
Income from own operations, Carlos Bakery P 3,000,000
Add: Income from Zeus
Share (2,500,000 × 60%) P 1,500,000
Add: Realized mark-up on beg. inventory
(20,000 × 90 × 50%/150% × 60%) 360,000 1,860,000
Consolidated net income P 4,860,000
1. A
2. C
3. B
4. C
5. C
6. B
7. D
8. C. P474,400
Cost of sales, Boboy P 380,000
Cost of sales, Homer 210,000
Less: Intercompany sales 2015 (120,000)
Mark-up beginning inventory ( 4,000)
Add: Mark-up on ending inventory 8,400
Consolidated cost of sales P 474,400
9. A. P1,000,000 and P690,000
Sales of Tomas Co. P 800,000
Sales of Badong Co. 300,000
Less: Intercompany sales ( 100,000)
Consolidated Sales P1,000,000
197
Cost of sales of Tomas Co. P 600,000
Cost of sales of Badong Co. 180,000
Less: Intercompany sales ( 100,000)
Add: Mark-up on ending inventory
(100,000 × 40%) × 40% 16,000
Consolidated cost of sales P 696,000
11. B. P4,860,000
Income from own operations, Carlos Bakery P 3,000,000
Add: Income from Zeus
Share (2,500,000 × 60%) P 1,500,000
Add: Realized mark-up on beg. inventory
(20,000 × 90 × 50%/150% × 60%) 360,000 1,860,000
Consolidated net income P 4,860,000
12. C. Overstated by P370,000
Intercompany receivable/payable P 120,000
Intercompany profit (750,000-500,000) 250,000
Consolidated current assets overstated by P 370,000
13. A. P1,360,000
Sales to non-affiliates 8,000 units
Gross profit per unit (220-50) × 170
Realized gross profit P1,360,000
14. D. P224,000
Net income reported by P, 2014 P 240,000
Less: Unrealized mark-up on ending inventory 2014
(180,000-130,000) × 40% × 80% ( 16,000)
Consolidated net income P 224,000
15. A. P161,600
Net income reported by P 2015 P 160,000
Add: Realized mark-up on beg. inventory
(180,000-130,000) × 40% × 80% 16,000
Less: Unrealized mark-up on ending inventory
(210,000-150,000) × 63/210 × 80% ( 14,400)
Consolidated net income P 161,600
16. D. P536,428
Cost of sales reported by Belgium (660,000/140%) P 471,428
Cost of sales reported by Hillinger (510,000/120%) 425,000
Less: Intercompany sales (140,000 ÷ 240,000) (380,000)
Add: Unrealized mark-up on ending inventory
Sales to Hillinger (downstream) 42,000 × 40%/140% 12,000
Sales to Belgium (upstream) 48,000 × 20%/120% 8,000
Consolidated cost of sales P 536,428
17. A. P67,600
198
Operating income of Belgium P 70,000
Add: Income (loss) from Hillinger
Share (20,000 x 80%) P 16,000
Less: Unrealized mark-up (downstream) (12,000)
Unrealized mark-up (upstream) 8,000 x 80% ( 6,240) ( 2,400)
Consolidated net income P 67,600
18. A. 15 years
January 1, 2012 to December 31, 2014 = 3 years
Depreciation per year = P60,000/3 years
= P20,000
Price paid by Blank to Grand P 276,000
Add: Debit to truck per eliminating entry 24,000
Original cost of the truck P 300,000
Depreciation per year ÷ 20,000
Original economic life on January 1, 2012 15 years
19. A. P870,000
2014 combined sales P 920,000
Less: 2014 intercomapny sales ( 50,000)
Consolidated sales P 870,000
20. A. P3,000
Selling price P 50,000
Less: Cost of sales 40,000
Original unrealized profit P 10,000
Unsold percentage 30%
Unrealized profit P 3,000
21. B
22. A. P1,025,000
Combined 2015 sales P 1,025,000
Less: 2015 intercompany sales 0
Consolidated sales P 1,025,000
23. D. P477,000
Combined cost of sales P 480,000
Less: 2015 intercompany sales 0
Less: Unrealized profit in the 2015 beg. inventory from 2014 ( 3,000)
Add: Unrealized profit in 2015 ending inventory 0
Consolidated cost of sales P 477,000
24. B. P76,250
Combined cost of sales P 160,000
Less: intercompany sales revenue ( 110,000)
Add: Unrealized profit taken out of inventory
(35,000 x 75%) 26,250
Consolidated cost of sales P 76,250
25. A. P54,250
199
(P115,000 x 70%) – P26,250 = P 54,250
26. A. P4,000
Selling price P 60,000
Less: Cost of sales ( 48,000)
Unrealized profit P 12,000
Unsold fraction 1/3
Credit inventory P 4,000
27. A. P720,000
P500,000 + 400,000 – 200,000 + 20,000 = P720,000
28. B. P76,000
(120,000 x 80%) – (200,000 x 20% x 50%) = P76,000
29. D. P24,000
Downstream situation.
30. A. overstated by P320
It will be overstated by the amount of the minority interests’ share of the P1,600 of
profit margin in the P9,600 of materials carried over to 2015 = 20% x P1,600 =
P320
31. C. P522,500
Grebe plus Swamp’s separate cost of goods sold = P400,000 + 320,000
= P720,000
Less: Intercompany sales (200,000)
Profit (12,500-10,000) 2,500
Consolidated COGS P522,500
32. B. P12,500
33. A.
34. A. P8,400
Squid’s reported income P 100,000
Less: Unrealized profits in the ending inventory ( 16,000)
Squid’s adjusted income P 84,000
Minority interest percentage 10%
Minority interest income P 8,400
35. B. P22,000
Unrealized profit in inventory:
P132,000 – (P132,000/1.2) = P22,000
36. A. P506,000
Income from Salt for 2015:
Share of Salt’s income (P880,000 x 60%) P 528,000
Less: Unrealized profit in ending inventory ( 22,000)
Income from Salt P 506,000
37. C. P664,000
200
Share of Cliff’s reported net income
P900,000 x 75% P 675,000
Add: Unrealized profit in beginning inventory 27,000
Less: Unrealized profit in ending inventory ( 38,000)
Income from Cliff P 664,000
38. A. P185,000
Sales (P620,000-160,000) P 460,000
Cost of goods sold (117,600)
Expenses (125,000)
Minority interest ( 32,400)
Consolidated net income P 185,000
39. A. P117,600
Combined cost of sales (210,000+72,000) P 282,000
Less: Intercompany sales (160,000)
Less: Unrealized profit in beginning inventory
[49,000 – (49,000/1.4)] ( 14,000)
Add: Unrealized profit in ending inventory
[33,600 – (33,600/1.4)] 9,600
Consolidated cost of goods sold P 117,600
40. B. P150,600
Lapwing separate income: P 100,000
Add: Realized profit in beg. inventory 6,000
Less: Unrealized profit in ending inventory ( 10,000)
Lapwing adjusted separate income P 96,000
201
Net Income - Attribution to Equity Holders of Parent/Controlling or
Parents Interest
1. D 6. A
2. D 7. C
3. C 8. B
4. B 9. D
5. D 10.C
11.B
Net Income-Parent P5,000,000
- (5,000)
Dividends (400,000)
12.C
Net Income-Parent P230,000
202
13.B
Net Income-Parent P230,000
14.B
Net Income-Parent P215,000
UP, beg. 0
15-17. B C & A
203
Net Income-
154,000 149,000 165,000
Subsidiary
UG (50,000) - -
RG - - 50,000
Consolidated net
P404,000 P433,000 P479,000
Income
Controlling Interest
P373,200 (15) P403,200 (16) P446,000 (17)
Attributable to Parent
18-19. A & D
2015 2016
UG (14,000) -
RG 2,000 2,000
Controlling Interest
P283,000 (18) P429,100 (19)
Attributable to Parent
20. B
204
Unrealized Mark up in ending Inventory (84,000/1.40)
(24,000)
x40%)
RG 2,000
21. B
22. D
23. A
205
Unrealized Mark up on ending inventory (210,000-
(14,400)
150,000) x 63/210 x 80%
24. A
25. C
26. B
206
Upstream- UPEI (91,200)
Downstream-RPBI 78,750
- 144,000
27. C
Gain 800,000
Upstream (4,000)
28. C.
Amortization 114,000
Downstream (360,000)
207
29. A
30. D
208
31. D
Downstream-RPBI 42,000
32. B
RG 2,000
209
Minority interest income 55,000
33. A
Goodwill 129,000
Upstream 45,000
34. C
35. B
36. A
37. B
38. A
39. D
40. A
210
Net Income
Non-Controlling Interest
1. B
2. C
3. A
4. B or D
5. B
6. D
7. C
8. A
9. B
10.C
11.D
Net Assets (1,100,000x20%) P220,000
NCI P233,000
12.C (350,000x20%)
13.C
Net Income of Pablo P90,000
Total P82,000
211
Minority percent 10%
14.B
Net Assets (100,000x25) P25,000
15-16 B & C
Sales P1,000,000
Expenses (400,000)
Total P350,000
17. A
Sales P150,000
212
Cost of Sales (110,000)
Total P10,000
Total P140,000
18-20 D A & B
Subsidiary’s Net
P154,000 P149,000 P165,000
Income
Non-controlling
P30,800 (18) P29,800 (19) P33,000 (20)
interest
21-22 B & A
2015 2016
Subsidiary’s Net
P150,000 P150,000
Income
Unsold Merchandise
8,000 -
(80,000/2 x 20%)
213
Minority percent 10% 10%
Minority interest
P14,200(21) P15,000 (22)
income
23-24 A & C
2015 2016
Subsidiary’s Net
P132,000 P197,000
Income
UG (14,000) -
Adjusted Net
P120,000 P199,000
Income
Minority interest
P12,000(23) P19,900 (24)
income
26. A
Total P212,000
27. A
RG (90,000x25%) 22,500
RG (180,000x25%) 45,000
28. C
29. D
215
Downstream Sale (14,000)
30. A
1,250
31. A.
Reserves 900,000
Total P1,900,000
32. C
216
Total P2,235,000
33-35. A B & D
Cost of investment in
(100,000)
Gemma
Non-Controlling
P36,000 (33) P55,200 (34) P91,200(35)
Share
36-38 A B & A
Cost of investment in
(30,000)
Fit
Fair Value
10,000 5,000
Adjustment
217
Minority Percent 20% 44%
Non-Controlling
P10,000 (36) P18,920 (37) P28,920 (38)
Share
39-40. B & A
Cost of investment in
(50,000)
Fit
Fair Value
46,550 27,550
Adjustment
Non-Controlling
P260,260 (39) P24,510 P50,770 (40)
Share
218
Net Income
Consolidated Group
1. D
Ordinary shares P4,500,000
2. D (850,000-80,000+8,000)
3. C (1,070,000-6,400-120,000+14,400)
4. C
Intercompany receivable/payable P120,000
5. A
Sales to non-affiliates 8000 units
219
Realized Gross Profit P1,360,000
6. C
Cost of Sales-Parent P380,000
7. A
Sales-Parent P800,000
Sales-Subsidiary 300,000
8. A
Cost of Sales-Parent P600,000
9. A
Sales-Parent P800,000
Sales-Subsidiary 300,000
220
Intercompany Sales (100,000)
10.C
Cost of Sales-Parent P600,000
11.C. Failure to eliminate the mark up on ending inventory will overstate the
consolidated ending inventory and will understate the cost of sales, thus,
understating the consolidated net income by P1,800. (60,000 x 30% x 10%)
12.D
Cost of Sales-Parent sold to outsiders (660,000- P371,428
140,000)/140%
13.A
Sales-Parent P3,850,000
Sales-Subsidiary 1,610,000
221
Intercompany Sales (480,000+437,5000) (917,500)
14.C
Sales-Parent P1,946,000
Sales-Subsidiary 1,176,000
15-1. C C B & A
Sales-Parent P21,500,000
Sales-Subsidiary 10,000,000
222
Unrealized Profit-ending Inventory (152,000+30,000) 330,750
Amortization 40,000
19. D
20. A
Goodwill 5,910,000
21. B
223
Fair Market Value- Identifiable Asset 19,050,000
Goodwill 5,238,000
Investment (15,480,000)
- (252,000)
22. A
23. D
24. A
Goodwill 522,000
224
Investment (1,458,000)
25. B
- (270,000)
Gain 351,000
APIC (19,800)
NCI 828,000
27. A
1,000,000/5 P200,000
500,000/10 50,000
28. A
1,000,000-800,000 P200,000
500,000-325,000 175,000
225
29. A
Sales-Parent P2,000,000
Sales-Subsidiary 1,000,000
30. B
31. A
32. B
226
Depreciation – 2019 (10,000)
33. A
34. D
35. B
36. A
37. C
227
Mark up on beginning inventory(100,000-75,000)x16% (4,000)
38. D
39. D
40. D
228
Retained Earnings/Common Share/Dividend
1. D
Retained Earnings beg. P12,000,000
Dividends (225,000)
2. D
3. A
Retained Earnings beg. P440,000
Dividends -
4. C
Retained Earnings beg. P3,150,000
Dividends (120,000)
229
Consolidated Retained Earnings-end P3,572,070
5. B
Retained Earnings beg. P4,860,000
Dividends -
6. C
Net Income reported by Entity A P1,00,000
Dividends (200,000)
Total P900,000
230
Elaine’s retained earnings P720,000
11.D
12.C
13.A
14.D
15.D
16.D
17.D
18.A
Retained earnings beg. P1,460,000
19-20 A & A
P Co. S Co.
21. D
231
Retained earnings beg. P485,000
22. A
23-24 C & A
Jewell Gem
Pre-acquisition retained
(45.000) (40,000)
Earnings
25. A
26-27. D & A
Body Fit
232
Pre-acquisition retained
(10.000,000) (6,000,000)
Earnings
28. C
29-30. A & C
Fusion Spine
233
Earnings
31. A
32-34 A B & A
Y Z W
‘million ‘million ‘million
35. A ‘million
234
w Co: share of post- acquisition profits60%x3 2
36. C
37. B
38. B
39. B
40. B
235
5. B Recoverable Amount P 265,000
Expense P 25,000
Dividends 8,400
Profit P 18,550
236
9. A Dividends P 3,150
Profit P 3,675
Dividends (5,250)
13. B
Dividends (30,000)
237
17. A 29. B
18. D 30. C
19. B 31. B
20. D 32. B
21. A 33. B
22. D 34. C
23. C 35. C
24. C 36. B
25. D 37. D
26. C 38. A
27. D 39. C
28. C 40. D
238
FOREIGN CURRENCY TRANSACTIONS
Without hedging activities (import, export, lending and borrowing
transactions)
3. D Principal P 328,000
Copyright 75,000
239
6. D Collection – Feb.18, 2018 $ 144,000
Receivable ( 132,000)
7. A
8. D
9. D
10. C
11. A
12. C
13. D
14. D
15. C
16. D
17. B
18. A
240
22. B
23. C
24. B
25. B
26. D
27. A
28. B
29. D
30. C
33. A
34. D
35. A
36. A
37. A
38. C
39. B
40. A
241
Hedges that not requires a Hedge Accounting
3. D
4. D
242
8. D Note Amount – Dec. 31, 17 $ 295,000
9. D
10. D
11. D
14. A
15. A
16. D
17. D
18. C
19. D
20. B
21. C
22. A
23. D
24. D
243
25. D
26. D
27. C
28. D
29. C
30. C
31. D
32. D
33. C
34. D
35. D
36. D
37. B
38. D
39. D
40. B
244
Hedges that not requires a Hedge Accounting
Speculation
245
9. B Ordered parts cost $ 100,000
10. C
11. B
14. C
15. C
246
Forex Loss per unit P .18
247
24. C
248
31. A Forward Rate – Oct. 12, 2017 P 3.15
Sales P 300,000
249
36. A Forward Rate – Dec. 12, 2017 P 3.15
40. A
250
Hedges that require a Hedge Accounting:
Multiple Choice
11 C 21 B 31 A
12 C 22 C 32 A
13 C 23 A 33 D
14 D 24 B 34 A
15 C 25 D 35 A
16 C 26 D 36 D
17 A 27 B 37 A
18 C 28 D 38 A
19 D 29 B 39 A
20 B 30 A 40 A
Problem-solving
Hedged Item/Commitment:
* 10/02/2017: Original forward rate (180 days) P 0.53
12/31/2017: Current (remaining) forward rate (90 days) 0.58
Forex loss per unit P 0.05
Multiplied by: Number of foreign currencies 1,000,000
Foreign exchange loss due to hedged item/commitment P 50,000
Hedge Instrument:
10/02/2017: Original forward rate (180 days) P 0.53
12/31/2017: Current (remaining) forward rate (90 days) 0.58
Forex loss per unit P 0.05
Multiplied by: Number of foreign currencies 1,000,000
Foreign exchange gain due to forward contract P 50,000
251
This hedge is called a “perfect” hedge and the hedge is considered to be highly
effective which satisfies the 80% - 125% acceptable range.
This criteria of 80% - 125% note under PFRS 9 was already removed
The forward rate generally differs from the spot rate, but as one moves closer to the
expiration date (or settlement date) the difference between the spot rate and the
forward rate for the remaining period of the contract becomes smaller and smaller so
that at the expiration date, the forward rate will have converged with the spot rate.
Protecting against an adverse change in the exchange rate between the order date
(commitment date) and the transaction date is hedging a firm foreign-currency
denominated commitment.
The entry to record the foreign exchange loss due to hedged item/commitment on
December 31, 2017 would be:
17. A
252
18. C
*If the date of transaction and date of settlement of forward contract falls on the same
date, obviously the spot rate and the current (remaining) forward rate should also be
the same.
20. B
PAS 39 / PFRS 9 states that “When an entity enters into firm commitment to acquire an
asset or assume a liability that results from the entity meeting the firm commitment
attributable to the hedged risk that was recognized in the balance sheet.”
The date of transaction (date of purchase and delivery of equipment) would be:
253
To remove the carrying amount of the firm commitment from the balance sheet and
adjust the initial carrying amount of the equipment that results from the firm
commitment.
Equipment 40,000
Or, alternatively:
Equipment 530,000
Note that the Foreign Currency Exchange Firm Commitment account of P40,000 (a
liability) is effectively transferred to asset acquired. The value of the equipment should
be the original forward rate on the October 2, 2017 (date of hedging). P0.53 (P0.53 x
1,000,000 = P530,000).
P1.30 spot rate (on delivery date/transaction date) x 200,000 baht = P260,000
The date of transaction (date of purchase and delivery of equipment) would be:
To remove the carrying amount of the firm commitment from the balance sheet and
adjust the initial carrying amount of the equipment that results from the firm
commitment.
254
Equipment 10,000
*The foreign currency exchange rate gain due to hedged item/commitment was
computed as follows:
If the date of transaction and date of settlement of forward contract falls on the same
date, obviously the spot rate and the current (remaining) forward rate should also be
the same.
23. A
Levin is a buyer of goods, and a seller of foreign currency under a forward contract.
Hedging is setting aside a fund to a bank or financial institution that is willing to absorb
any gain or loss resulting from a hedged transaction.
It is called a hedged transaction because, no matter what the spot rates are, the buyer
(or seller) who made a hedged contract, he would be paying (or receiving) the stated
amount in the hedge contract.
FC Receivable FC Payable
(¥400,400 x P0.55) 220,220
(¥400,400 x P0.50) 200,200
24. B
The focus was the forward contract, which was entered into on 12/12, the forward rate
was 0.60.
255
The report period being asked was 12/31, the forward rate was 0.63.
The transaction is a purchase, so the hedging instrument was a receivable. The value
increased, so it was a gain.
25. D
C - Zero, it was only the transaction date. No changes yet on the forex.
Forward
Payable Contract
@ spot rate Receivable
Oct. 1 43 44.00
Dec. 31 47 Loss 4 46.00 Gain 2 Net loss 2
Jan. 30 50 Loss 3 50.00 Gain 4 Net gain 1
(on settlement
date)
x 93750
The rate used in the forward contract on Jan. 30 (settlement date) is actually the spot
rate. It just so happens that the spot rate and the forward rate are both 50. Just keep
in mind that it's the spot rate to be used in there.
26. D
Forex gain or loss on firm commitment on Dec. 31 (46.70 minus 44.30) x 15750 =
37,800
S Company is a seller, from that transaction it will have a receivable. The value of forex
using the forward rates increased, so it's a gain.
256
27. B
28. D
Spot rate
12/1/2013 (P6.01 x 1,000,000 Franc) P 6,010,000
12/31/2013 (P6.16 x 1,000,000 Franc) 6,160,000 P (150,000)
60-day futures
12/1/2013 (P6.06 x 1,000,000 Franc) P 606,000
12/31/2013 (P6.07 x 1,000,000 Franc) 6,070,000 10,000
Net forex gain (loss) P (140,000)
29. B
31. A
257
Foreign currency translation gain (loss) should be included in the Profit and Loss.
32. A
June 30 [400,000 FC x (P1.381 - P1.370)] P 4,400
July 31 [400,000 FC x (P1.385 - P1.381)] 1,600
Net forex loss P 6,000
33. D
June 30
(P2,600 - P1,400) P 1,200
July 31
[400,000 - (P1.385 - P1.375) - P2,600] 1,400
Net gain P 2,600
34. A
Down payment
(50,000 FC x P1.350) P 67,500
Balance due
(400,000 x P1.370) 548,000
Cost of Machinery P 615,500
35. A
36. D
Forward
Contract Option
Gain (loss) commitment
[100,000 FC x (P1.250 - P1.320)] P (7,000) P (7,000)
Gain on hedging instrument
[100,000 FC x (P1.320 - P1.250 )] 7,000 7,000
Gain (loss) excluded from hedge effectiveness
[100,000 FC x (P1.270 - P1.250 )] (2,000)
Premium paid is on time value (2,100)
Loss effect on earnings P (2,000) P (2,100)
258
37. A
38. A
39. A
40. A
259
Hedges that requires a Hedge Accounting
Multiple Choices
1 A 11 A 21 B 31 A
2 D 12 B 22 C 32 A
3 B 13 D 23 B 33 A
4 D 14 A 24 C 34 A
5 C 15 D 25 D 35 D
6 B 16 B 26 A 36 B
7 E 17 A 27 C 37 C
8 B 18 B 28 C 38 B
9 C 19 D 29 D 39 B
10 C 20 A 30 D 40 B
Problem-solving
4. B
Incidentally, the entries to record on:
To remove the gain recognized in other comprehensive income and the initial
carrying amount of the equipment that results from the hedged transaction by
this amount.
260
Other Comprehensive Income – gain 40,000*
Equipment 40,000
*If the date of transaction and date of settlement of forward contract fall on the
same date, obviously the spot rate and the current (remaining) forward rate
should also be the same.
Or, alternatively:
Equipment 530,000
Equipment 530,000
Which reflects the starting presumption, i.e. that Asser Tamayo, Inc. had
effectively fixed the purchase price of its machine at P530,000. However, the
route to get to this position may also seem slightly convulated.
5. A
261
Receivable Futures contract
@ spot rate payable
Nov. 1 51.30 50.50
Dec. 31 52.60 Gain 1.3 52.40 Loss 1.9 Net loss 0.6
Jan. 31 51.80 Loss 0.8 51.80 Gain 0.6 Net loss
(0.2 x 315000)
63000
The rate to be used in the futures contract is 90-day futures on Nov. 1, since there are
90 days before settlement date; 30-day futures on Dec. 31, since there are 30 days
before settlement, and spot rate on settlement date.
6. B
Forex gain or loss on the forward contract on Feb 28 (62.05 minus 62.35) x
625000 = 187500
SBC company is the buyer, it has a payable. The forward contract would then be
a receivable. As such, the decrease in rates would mean a loss.
8. A
$.04 × C$150,000 = $6,000
9. B
$.03 × C$150,000 = $4,500
10.C
Strike Price $.97 × C$150,000 = $145,500
11.B
12.C
13.D
262
[$1.41 - $1.37 = $.04 × €400,000 = $16,000 Discount] & [$1.41 - $1.36 = $.05
× €400,000 = $20,000 Adjustment at Delivery]
14.A
P5,000,000
15.C
Variable rate on Jan. 1, 2016 0.01
Underlying interest rate 0.06
Difference 0.04
16.C
17.D
18. D
Variable rate on Jan. 1, 2017 0.07
Underlying interest rate 0.10
Difference 0.03
20.A
Market Price, Dec. 31, 2015 P 110
Underlying Price 100
Difference 10
Payment for call option 50,000
Net cash settlement P 500,000
21.A
Market Price, July 1, 2016 P 115
Underlying Price 100
Difference 15
Payment for call option 50,000
Net cash settlement P 750,000
22.C
Call option is not exercise because the market price on July 1, 2016 is lower than
the call price.
23.D
24.B
Australian lobster - kilos 8,000
multiplied by price per kilo P 1,200
Notional Value of forward contract P 9,600,000
25.C
Market Price, Dec. 31, 2015 P 1,500
Underlying Price 1,200
Difference P 300
27. B
Increase in Fair value:
Fair value of call option P 2,500,000
(500,000 bushels x P5)
Payment for call option (100,000) P 2,400,000
Decrease in Fair value:
Fair value of call option
(500,000 bushels x P4) P 2,000,000
Call option, Dec. 31, 2015 2,500,000 500,000
Gain on call option P 1,900,000
28.B
Purchase milk
(50,000 units x P9) P 450,000
Purchase ice cream
(30,000 units x P25) (750,000)
Purchased sugar
(20,000 units x P15) 300,000
Derivative asset P 600,000
265
Hedges that requires a Hedge Accounting
Multiple Choices
1 D 11 D 21 B 31 B
2 D 12 D 22 C 32 A
3 E 13 D 23 A 33 C
4 D 14 C 24 A 34 B
5 D 15 C 25 A 35 A
6 B 16 A 26 A 36 A
7 B 17 B 27 C 37 A
8 C 18 A 28 D 38 A
9 C 19 A 29 A 39 A
10 C 20 A 30 D 40 A
Problem-solving
1. A
The loss on the forward contract is reported in other comprehensive income.
2. B
The total gain on the forward contract is ($1.30 - $1.26) x €100,000 = $4,000.
Changes in the value of the forward are reported in other comprehensive income
until the hedged forecasted transaction is reported in income. In this case, the
forecasted transaction results in sales revenue, reported in 2011.
3. C
Changes in the value of the forward contract remain in other comprehensive
income until the merchandise is sold. The merchandise is reported at the spot
rate at the date of purchase, $1.42.
4. A
Changes in the value of the forward are reported in other comprehensive
income. The $100 loss on the payable is exactly offset by a reclassification of
$100 out of other comprehensive income, so there is no net effect on income.
5. A
At the end of the year, other comprehensive income has a credit balance of
$100. When the merchandise is sold, it is reclassified as a reduction in cost of
266
goods sold; $14,100,000 = $14,200,000 - $100,000.
6. A
The equipment is recorded at the spot rate of $1.35 x €100,000 = $135,000,
adjusted for the $6,000 [= $1.35 - $1.29) x €100,000] gain on the forward
contract.
7. A
Cost of the Option Contract $4,000
8. C
$2.17 - $2.13 = $.04 × £250,000 = $10,000 Positive Adjustment
9. D
Cost of the Option Contract $5,000
10.A
11.D
12.B
Option Expense is the Balance Sheet Fair Value of the Option for 2014 = $1,600
13.A
$2.01 - $2.00 = $.01 × £100,000 = $1,000 Adjustment to AOCI for 2014
14.C
£100,000 × $2.00 Strike Price = $200,000 + $1,000 AOCI Adjustment =
$201,000 COGS
15.B
[£100,000 × $2.00 Strike Price = $200,000] + [$1,600 Fair Value of the Option
in 2014] = $201,600
16.A
FC Receivable from Exchange Dealer
(500,000 euros x $1.01) 505,000
Dollars Payable to Exchange Dealer 505,000
17.A
Foreign exchange loss – Other Comprehensive
Income (Balance Sheet) 10,000
FC Receivable to Exchange Dealer 10,000
To record a gain on the change in forward contract (500,000 x ($1.01-$0.99))
267
18.A
Foreign exchange loss – Other Comprehensive
Income (Balance Sheet) 5,000
FC Receivable to Exchange Dealer 5,000
To adjust the forward contract to its market value of $20,000.
The change in value of the forward contract [($.99 12/31 spot rate less $0.98
January 31, 2004 spot rate) x 500,000 euros] is $5,000.
19.A
Investment in FC (500,000 euros) 490,000
Dollars Payable to Exchange Dealer 505,000
FC Receivable from Exchange Dealer 490,000
Cash 505,000
To settle with the trader
20.A
21.A
Suppose that in February, the inventory is sold for $600,000. The entries to
record the sale and to reclassify the amounts from Other Comprehensive Income
(a $15,000 loss, including $10,000 loss at December 31, 2003 plus the $5,000
additional loss at January 31, 2004) into earnings are as follows:
Cash 600,000
Cost of goods sold 490,000
Sales 600,000
Inventory 490,000
To record the sale
268
Hedges that requires a Hedge Accounting
1 C 11 D 21 D 31 D
2 B 12 A 22 A 32 C
3 B 13 D 23 A 33 B
4 A 14 C 24 A 34 B
5 B 15 A 25 A 35 D
6 B 16 D 26 A 36 C
7 C 17 C 27 A 37 A
8 A 18 A 28 D 38 A
9 A 19 A 29 A 39 B
10 A 20 C 30 A 40 B
Problem-solving
1. A
2012 2013
P (6,000,000) P 9,093,750
2. A
No. of rupee 25,000
Multiplied by exchange rate P 1.24
Income from subsidiary P 31,000
3. D
5. B
6. B
7. D
270
8. C
NZ dollar rate Peso
Net Assets, Jan. 1, 2013 20,000 P 15 P 30,000
Net Income 10,000 19 190,000
Net Assets, Dec. 21, 2013 30,000 490,000
Net Assets at current rate 30,000 4 630,000
Translation adjustment - P 140,000
9. A
Exchange
Yen Peso
rate
Net Assets Beg. 200,000 0.44 88,000
Net Income 200,000 0.46 92,000
Net Assets translated at rate
During the year 400,000 180,000
At the end of the year 400,000 0.48 192,000
OCI - translation adjustment - credit - (12,000)
10.A
11.B
$3,800 value of the expected sale
12.B
Converted value of Pinco's long-term debt $ 42,000
translated value of Sinco's balance sheet (40,000)
Amount should be recognized in current income $ 2,000
271
FOREIGN CURRENCY TRANSACTIONS
1. B
2. C
3. D
4. F
5. C
6. B
7. B
8. A
Cr Equity CU 400,000
9. A
10.C
11.B
12.C
13.A
14.D
15.D
16.D
17.A
18.C
19.B
272
February 250,000 71.03% 1.7103 427,585
Total 4,178,246
*[(1+5%)^12) – 1} = 0.7959
20.B
Land CU 240,000
Gain on net monetary position CU 240,000
To recognized land restatement
(CU 300,000 x 80%)
21.B
Revenue CU 80,000
273
Loss on net monetary position 40,000
22.A
23.A
24.B
274
Foreign exchange loss CU ( 789)
25.A
26.C
27.A
28.D
275
Divided by average spot rate on 20X1 CU 2.3
29.A
Multiply by 12 months
30.C
Foreign exchange gain 18,480
Accrued interest on bank loan 11,282
Bank loan 208,333
Cash 238,095
To recognise foreign exchange gain on settlement of the foreign bank loan (FCU
500,000 ÷ 2.1)
31.D
32.B
33.D
34.B
35.C
36.D
37.B
38.B
39.B
40.D
276
Translation from a functional currency to the presentation currency
(closing/ current rate method)
1. C
Rent 200,000
2. D
277
Rent 200,000
17.D
18.D
19.C
20.D
21.C
22.D
23.B
24.C
25.A
26.B
3. A
27.D
4. B
28.C
5. A
29.B
6. A
30.D
7. A
31.A
8. B
32.C
9. D
33.B
10.B
34.A
11.A
35.D
12.D
36.A
13.B
37.B
14.B
38.A
15.C
39.D
16.D
40.A
278
Remeasurement from a foreign currency to the functional
currency (temporal method
1. A
2. D
Inventory at FC 100,000
3. C
279
Marketable equity securities at Peso P19,000
Inventory at FC 100,000
4. B
5. B
6. A
7. C
8. D
9. B
10.D
11.D
12.D
13.C
Total P 505,000
280
Adjusted retained earnings 475,000 (1,585,000)
14.A
Sales 90,000
Depreciation (1,500)
15.B
281
Cumulative translation adjustment – 2020 (credit) P 50,000
16.A
282
Debit in balance sheet
17.A
18.B
19.C
20.B
21.D
22.C
283
Purchases 300,000
23.C
24.A
25.B
Accounts receivable 2,000
Foreign exchange gain 2,000
[$20,000 x (46.70 – 46.60)]
26.B
27.D
28.B
29.C
284
Transaction 1 [LCU 80,000 x (P 5.15 – P4.86)] (debit) P (23,200)
30.A
11/16 – NO ENTRY
12/16 – Accounts Receivable 325,000
Sales 325,000
(10,000 x P 32.50)
12/31 – Accounts receivable 10,000
Foreign exchange gain 10,000
[10,000 x (P33.50 – P 32.50)
1/15 – Cash 330,000
Foreign exchange loss 5,000
Accounts receivable 335,000
(10,000 x P 33.00)
31.A
32.C
Sales P 325,000
33.D
See entries in number 31
34.A
285
Multiply by selling spot rate at December 31, 2013 P 51.00
35.B
36.D
Foreign Exchange loss 10,000
Accounts payable 10,000
37.D
Equipment 500,000
Accounts payable 500,000
38.A
Accounts payable 510,000
Cash 505,000
Foreign exchange gain 5,000
39.A
40.B
286
TRANSLATION OF FOREIGN CURRENCY FINANCIAL STATEMENTS
1. A
287
2. B
3. C
4. D
Balance P120
5. A
7. B
6. A 8. D
288
9. A 20.A
10.D 21.C
11.B 22.A
12.B 23.A
13.A 24.C
14.B 25.A
15.D 26.B
16.D 27.D
17.D 28.B
18.C 29.B
19.B 30.C
31.D
Translated at current
rate
P
Total 450,000
32.A
Depreciation H$ 12,000
Rent 20,000
289
Solution for 33 to 38
290
Income statement and Retained earnings Dollars Pesos
291
33.C
34.A
35.B
36.A
37.A
38.C
39.A
40.A
1. B
2. D
3. B
292
4. D
5. C
6. D- Pledges are recognized in income in the period in which they are receivable. One-
half of the P100,000 is receivable in 2011, or P50,000, 20% of which, P10,000, is
uncollectible resulting in net contributions to be recognized in the amount of P40,000 in
2011
7. C – The testator had stipulated that the P200,000 certificate be held to maturity and
that interest revenue be used to finance salaries for the pre-school program. As a
result, the P16,000 in interest income would be considered restricted to that purpose.
Since the testator had placed no other restrictions on the use of bequest, the remaining
amounts are unrestricted. An action by the board of trustees designating that P40,000
be used for the future purchase of the equipment for the pre-school program would be
considered a designation and the P40,000 would be reported as unrestricted funds
designated for the pre-school program.
9. D – According to FASB 116, donor restricted contributions are revenues in the year
the contribution is made, not in the year contribution is spent. According to FASB 117,
contributions which are restricted temporarily should be reported on the statement of
activities as temporarily restricted revenues.
10. C – According to FASB 116 gifts long lives assets should be reported as unrestricted
support if the organization has an accounting policy which does not imply a time
restriction on such gifts
293
11. B - According to FASB 116, contributions are reported as revenue in the year
received, whether the donors place time or use restrictions on the resources. According
to FASB 117, net assets should be disclosed according to whether they are unrestricted,
temporarily restricted net assets for the year ending December 31, 2017
Total 35,000
13. D - According to FASB 116, a multiyear pledge should be reported at its present
value. According to FASB 117, if there is a time restriction on the pledge, the pledge
should be reported as temporarily restricted revenue in the year the pledge is given. In
Faith Haven's situation the pledge should be reported as temporarily restricted revenue
in 2011. The pledge should be reported at its present value. This amount is calculated
by using teh present value of an ordinary annuity factor for 5 periods at 6%: P1,000 x
4.212376 = P4,212 rounded.
15. D 17. B
16. B 18. A
294
19. D 26. A
20. A 27. B
21. A 28. C
22. B 29. A
23. A 30. B
24. A 31. C
25. D
32. B- Under AICPA FASB SFAS 117, reclassifications result from expiration of donor-
imposed restriction.
33. B
34. D
35. A
36. D
37. D
38. D
39. C
40. D- Pledges are recognized in income in the period in which they are receivable.
One-half of the P100,000 is receivable in 2011, or P50,000, 20% of which, P10,000, is
uncollectible resulting in net contributions to be recognized in the amount of P40,000 in
2011
295
NOT FOR PROFIT ORGANIZATIONS
2. Required: The item(s), if any, that are ongoing or central transactions for a not-for-
profit hospital.
(B) Revenue from health care services include inpatient and outpatient services
provided directly to patients for their medical care. The resulting revenues derive from
furnishing room and board and nursing services. Health care service revenues are also
earned by the operating room, recovery room, labor and delivery room, and other
ancillary departments that give patient care.
Answers (A), (C), and (D) are incorrect because room and board fees from
patients and recovery room fees are ongoing or central transactions.
296
4. Required: The definition of restricted net assets.
(D) In health care organization accounting, the term “restricted” is used to describe
resources that have been restricted as to their use by the donors or grantors of those
resources. Temporarily restricted net assets are those donor-restricted net assets that
can be used by the not-for-profit organization for their specified purpose once the
donor’s restriction is met. Permanently restricted net assets (for example, endowment
funds) are those with donor restrictions that do not expire with the passage of time and
cannot be removed by any actions taken by the entity.
Answer (A) is incorrect because donor restrictions are not removable by the
board. Temporary restrictions expire by passage of time or by actions by the entity
consistent with the donor’s restrictions. Answer (B) is incorrect because board-
designated restrictions are board-removable. Answer (C) is incorrect because income
generated by restricted net assets can be restricted for specific purposes.
(D) Contributions of noncash assets that are not long-lived are reported at fair value in
the statement of operations. Donated medicines, office supplies, and other materials
that normally would be purchased by a hospital should be credited at fair value as other
revenue because they directly relate to ongoing major operation but are not derived
from services directly provided to patients.
Answer (A) is incorrect because donated assets should be recorded at their fair
value when received. Answers (B) and (C) are incorrect because this donation should
be credited to another revenue account or a gain account.
7. Required: The true statement about external reporting by a health care organization.
297
(B) The basic financial statements of a health are organization include a balance sheet,
a statement of operations, a statement of changes in equity or net assets, and a
statement of cash flows.
Answer (A) is incorrect because fund accounting may be used for internal
purposes but is not required or encouraged for external reporting. Answer (C) is
incorrect because the statement of changes in equity or net assts may be combined
with the statement of operations. Answer (D) is incorrect because the statement of
operations all HCOs, including NPOs, should report a performance indicator and other
changes in net assets.
9. C
10. B
11. C- Health Care Organizations, provides that for contractual adjustments and
discounts is recognized on the accrual basis and deducted from gross patient service
revenue to determine net patient revenue. Bad debts expense is reported as an
operating expense, not as a contra to gross patient service revenue.
Thus:
Gross patient service revenue 980,000 Contractual adjustments (115,000)
Allowance for discounts - employees (15,000) Net Patient Service Revenue
850,000
12. B
13. B
14. B
15. B- Patient service revenues are recognized at established rates on an accrual basis,
P8, 000,000. The discharge method is not acceptable according to generally accepted
accounting principles.
298
16. B - Donated supplies are recorded by the hospital as a debit to supplies and a
credit to other operating revenues of P10,000. Supplies is directly used for hospitals
principal operations.
17. C- An unrestricted bequest is a revenue derived from some source other than
operations and would be recorded as a non-operating revenue of P100,000.
18. D- Since Cebu Hospital does not have control over the endowment fund and will
only receive income earned on the P100,000, Cebu Hospital would make no entry to
record the establishment of the endowment. A memo entry would be sufficient to
enable the endowment to be disclosed.
19. A- The unrestricted bequest would be accounted for as non-operating revenue. The
amount to be recognized will be the fair value of the stock, regardless of the testator's
basis.
20. C- In 2011, the investments would have been written down from P300,000 to
P290,000 resulting in a valuation allowance of P10,000. In 2012, they would have been
further written down to P250,000 with an increase in the valuation allowance of
P40,000.
22. A- Gifts, grants, and bequests that are received by a hospital and are not restricted
by donors are reported in the statement of revenues and expenses as non-operating
revenues.
299
23. D- Endowment resources, including pure endowment funds in which the principal
may not be expended, and term endowment funds in which principal may be expended
upon release of the prohibition on such expenditure, are accounted for as restricted
funds when they are received.
24. D- Since the P10,000 donation is being held in permanent trust by the bank, it is
not an asset of the hospital and would not be recorded in the accounting records. it
would, however, be disclosed.
25. C - Since the investment is carried on the books at P50,000 in the investments
account, a a-edit is necessary to that account to remove the investment. Capital gains
and losses are treated as adjustments to principal, not as income, to the credit for the
P70,000 goes to the endowment fund balance, not to revenue.
26. D- When a fixed asset, such as land, is donated to a hospital without any
restrictions, the value is generally added directly to the fund balance of the general
fund. The land will be recorded at its fair value, P400,000. Since it is subject to a
liability in this case, the mortgage payable will be recorded in the amount of P150,000.
The net amount of P250,000 will be added to the fund balance.
27. A- When goods which a hospital would otherwise have to purchase are donated,
they are reported as an operating expense and simultaneously as other operating
revenues before to No. 17 for further discussion.
28. A - The AICPA Audit and Accounting Guide. Health Care Organizations, provides
that for contractual adjustments and discounts is recognized on the accrual basis and
deducted from gross patient service revenue to determine net patient revenue. Bad
debts expense is reported as an operating expense, not as a contra to gross patient
service revenue. Thus:
300
Less: Contractual adjustments 100,000
29. D- The AICPA Audit and Accounting Guide, Health Core Organizations, charity care
does not qualify for recognition as receivables or revenue in the financial statements.
According to the AICPA Audit and Accounting Guide, Health Care Organizations
management's policy for providing charity care, as well as the level of charity care
provided, should be disclosed in the financial statements.
30. C- According to the AICPA Audit and Accounting Guide, Health Care Organizations,
a hospital's other revenue, gains, and losses are derived from services other than
providing health care services or coverage to patients. Other revenue, gains, and losses
typically include interest and dividends which are unrestricted as well as proceeds from
sales at gift shops and snack bars. Cash contributions from donors which are restricted
to the acquisition of capital assets during 2012 are not reported on the statement of
operations for 2011. The capital contribution should be reported on the statement of
changes in net assets for the year ended December 31, 2011 as an increase in
temporarily restricted net assets.
32. A- Cash flows from operating activities would include both the cash received from
patient service revenue of P300,000 and the cash received from gift shop sales of
P25,000. According to FASB No. 117, cash received from investment income that is
restricted by donors for the acquisition of long-lived plant assets should be reported as
financing activities.
301
33. B- The P750,000 and P50,000 are classified as part of operating activities since,
they are not restricted for any purpose. Refer to Nos. 8 and 33 for further discussion.
34. B
35. C- The P250,000 gift shop revenue is unrestricted revenue because the governing
board has control of this revenue. Thus:
P300,000
P250,000 (c)
37. C- A hospital prepares four statements comparable to the balance sheet, income
statement, retained earnings statement, and statement of cash flows prepared by a
business enterprise. They include the balance sheet, the statement of operations, the
statement of changes in net assets, and the statement of cash flows.
38. A- A hospital must segregate items that are unrestricted from those that are
restricted. In addition, the balance sheet must provide information about liquidity, by
either sequencing assets according to the nearness of their conversion to cash or
classifying them as current and noncurrent.
39. A- Property, plant and equipment of a not-for-profit hospital is part of the hospital's
unrestricted funds.
302
40. C- Fixed asset contributions are added directly to the unrestricted net assets
balance of a hospital. They are normally used for general operations.
1. D
2. B
3. A
4. C
5. A
6. C
7. A
8. B
COMPUTATION:
Assets P 5,000,000
Less: Liabilities (including deferred revenues of P 100,000) P 3,000,000
Fund Balance P 2,000,000
9. C - All items presented would be included in plant funds at July 31, 2009
10. B
Computation:
303
Income on unrestricted endowment received P 300,000
Income on restricted endowment expended P 75,000
Total endowment income P 375,000
11. A –
12. C- Regular or Pure endowment Funds are those whose principal has been specified
by the donor as nonexpendable therefore, the amount to classify as regular
endowment would be P500, 000.
13. C
14. B- The net asset balance is equal to assets minus liabilities or P5,000,000 -
P3,000,000 = P2,000,000. Deferred revenues are considered liabilities and are properly
included.
15. B- The university will recognize the total amount of tuition and fees earned net of
cancellations of P300,000 - 7,000 = P293,000. The P3,000 of tuition remissions granted
to faculty members' families will be treated as an expense item.
16. D- Revenues include the tuition assessed less any refunds, or P1,700,000. The
scholarships of P150,000 and the tuition remissions of P50,000 are treated as
expenditures and classified as student aid.
Current fund revenues will include all unrestricted gifts, P500,000, other unrestricted
resources earned during the period, and restricted current funds to the extent that they
have been expended for current operating purposes, P200,000. Current fund revenues
do not include restricted current funds received but not expended, nor resources which
304
are restricted by outside parties to loan funds, endowment or term endowment funds,
annuity and life income funds, plant funds, or agency
18. C- Restricted gifts are recognized as revenues in the epriod in which they are
expended. Unrestricted gifts are recognized as revenues whether they are expended or
not. As a result, the expended restricted gifts of P100,000 will be recognized as
revenues as well as the expended unrestricted gifts of P600,000 and the unexpended
unrestricted gifts of P75,000 for a total of P100,000 + P600,000 + P75,000 = P775,000.
19. C- A regular endowment fund is one in which the amounts contributed may not be
spent, but the earnings are available to the college or university, which would be the
case with the nonexpendable endowment of P500,000. Since the P300,000 amount is
ultimately expendable and the P100,000 is an amount designated from current funds,
neither would be a regular endowment.
20. A- Since the trust fund is being administered by a separate trustee, the college will
not account for it. As interest is remitted to the college, it will be reported with a debit
to cash. The amount is restricted by the donor to be used for the specific purpose of
supporting student scholarships and, as a result will be considered restricted current
funds. It will be recorded as deferred revenue since the amount received is not to be
reported as revenues until it is spent.
21. D- When funds have been designated for a purpose by the trustees, rather than
being legally restricted, they are accounted for in a quasi-endowment fund. The
P100,000 retained and invested for scholarship grants would be included in the fund
balance. In addition, other amounts retained, such as the P6,000 in ecimigs which had
not been disbursed as of 12/31/2011, would also be included in the fund balance for a
total of P106,000.
22. B- The P5,000,000 contributions from alumni for the science building should be
reported as temporary restricted revenue since it is restricted as to its purpose.
The P50,000 earnings will be used for scholarship and should be reported as temporary
restricted since it is restricted as to its purpose.
305
assets released from restrictions of P15,000 are reported as a positive amount for
unrestricted net assets for 2012. The P15,000 of travel expense is reported on the
statement of activities as an expense for 2012. According to FASB 117, all expenses are
reported on the statement of activities as decreases in unrestricted net assets. This
means that the use of the donation for faculty travel had no effect on unrestricted net
assets in 2012. Note that, when the donation was received in 2011, temporarily
restricted net assets increased by P15,000 on the statement of activities prepared for
2011.
24. A- Since the P25,000 contribution is intended for a purpose (purpose restriction),
therefore, it is classified as a temporarily restricted revenues.
26. D- The P500,000 and P100,000 are classified as unrestricted revenues and
therefore, they are port of operating activities in the statement of cash flows.
27. C- The current funds group of a not-for-profit private university include the current
restricted fund and the current unrestricted fund. Annuity funds and Loan funds are
different types of funds and are riot part of the current funds group.
28. D The plant fund of a university accounts for plant assets in the investment in plant
funds, and for funds restricted for future expenditures in the unexpended plant funds.
Additionally, funds for the retirement of indebtedness and funds for renewals and
replacements may be created.
29. D- A not-for-profit private university will typically have a current fund, a plant fund,
loan funds, endowment funds, and agency funds.
30. D- Loan funds consist of loans made to students, faculty, or staff. Life income funds
are funds contributed to a university requiring that the income be paid to a beneficiary
with the principal restricted for some time after which it becomes available to the
institution. Both types of funds may be encountered in a not-for-profit university.
31. C- Amounts received by a college which can be spent, but only in specific ways, are
included in restricted current funds. This would include dividends received from
securities which must be spent on faculty travel. 1 he contribution of securities which
must be held in perpetuity will take place in endowment funds.
306
32. A- College plant funds include unexpended plant funds, funds for renewals and
replacements, funds for retirement of indebtedness, and investment in plant. Restricted
current funds do riot include funds related to plant.
33. A- An endowment fund is used to account for funds received from donors where the
principal is nonexpendable.
34. B- A loan fund of a college is established for resources that are to be loaned to
students, faculty, or staff it is not for loans, notes, or bonds payable to others.
35. A
36. D
37. C
38. C
39. B
40. A
307
NOT FOR PROFIT ORGANIZATIONS
1. C
2. A
308
3. A
4. C
5. A
REQUIRED: The asset held by a nonprofit organization for which depreciation should
be recognized.
DISCUSSION: (A) SFAS 93, Recognition of Depreciation by Not-for-Profit
Organizations, requires all nonprofit organization to recognize the cost of using up
long-lived tangible assets (depreciation) in their general purpose external financial
statements. Hence, a building used for religious activity is ordinarily depreciable.
309
Answers (B) and (C) are incorrect because depreciation does not have to be
recognized for certain works of art and historical treasures whose economic benefit
or service potential is used up so slowly that their estimated useful lives are
extraordinarily long. Answer (D) is incorrect because land is normally not
depreciated by any organization.
6. D-
REQUIRED: The reporting of a transfer to an NPO with a direction that the assets
be used to aid a specific beneficiary.
DISCUSSION: (D) SFAS 136, Transfer of Assets to a Not-for-Profit Organization or
Charitable Trust that Raises or Holds Contributions for Others, applies when a donor
makes a contribution to a recipient entity that agrees either to user the assets for
the benefit of another entity designated by the donor or to transfer the assets of the
beneficiary. The recipient entity should recognize the receipt of the assets as a
contribution if the donor explicitly grants the entity variance power to redirect the
use of the assets or if the recipient and the beneficiary are financially interrelated.
However, if neither of these conditions applies, the recipient entity should recognize
the fair value of the assets as a liability/
Answers (A), (B), and (C) are incorrect because the recipient has not been granted
variance power, and the recipient and beneficiary are not financially interrelated
organizations. Thus, the transfer should be accounted for as a liability.
7. B
310
Answers (A), (C), and (D) are incorrect because, if the proceeds are used to
support general museum activities, repair existing collections, or purchase buildings
to house collections, the contribution must be recognized.
8. D
9. C
Answers (A) and (B) are incorrect because receipt of the promise is not sufficient for
recognition of a contribution. Answer (D) is incorrect because the possibility that
the condition will not be met must be remote before a contribution is recognized
10. C
311
expense in the period of receipt and use. The revenue and expense should be
measured at estimated fair value. This estimate can be obtained from the rate
schedule used by the utility company to determine rates charged to a similar
customer.
Answers (A), (B), and (D) are incorrect because the simultaneous receipt and use of
electricity should be recorded as an unrestricted revenue and an expense in the
period of receipt and use
11. C
12. D
312
Answers (A), (B), and (C) are incorrect because investment income may be reported
as an increase in either unrestricted or temporarily restricted net assets in these
circumstances.
13. B
14. B
313
Answers (A), (C), and (D) are incorrect because the donor-imposed restriction is
temporary. It will expire when the income is expended. Moreover, the income
cannot be classified as unrestricted because recognition and the expiration of the
restriction do not occur in the same period
15. B
16. C
17. B
314
REQUIRED: The correct accounting treatment of combined fund-raising and
educational materials or program services costs.
DISCUSSION: (B) When fund-raising costs are combined with program services
costs or educational materials, the total of these combined services should be
systematically and rationally allocated between the programs and fund-raising.
Answer (A) and (C) are incorrect because costs that do not completely relate to one
category should be allocated. Answer (D) is incorrect because the costs must be
allocated to the proper programs to which they relate.
18. C
19. B
REQUIRED: The expense classification for management and general expenses in the
statement of activities.
DISCUSSION: (B) Two functional categories of expenses for an NPO are program
services and supporting services expenses. Supporting services expenses, which do
not relate to the primary mission of the organization, may be further subdivided into
(1) management and general expenses, (2) fund-raising expenses, and (3)
membership development costs.
Answer (A) is incorrect because a direct reduction of fund balance would be the
result of a transfer or a refund to a donor. Moreover, fund accounting
information is not required to be externally reported. Answer (C) is incorrect
because program services expenses related directly to the primary mission of the
NPO. Answer (D) is incorrect because only costs directly related to a certain
315
source of support, such as a special event or estimated uncollectible pledges,
may be offset against revenue.
20. A
21. B
22. A
23. C
316
24. C
25. (c) Funds are considered to be restricted funds only when they are restricted by
donors or others as to the specific purposes for which the funds are to be used. This
would be the case of the P200,000 bequest. When funds are designated for a specific
purpose by the Board of Trustees or some other internal body are considered
unrestricted funds as is the case with-the P100,000 designated for college scholarships.
26. (a) A gift of donated property is recognized at its fair market value on the date of
the gift. Depreciation is recorded over its useful life. As a result, an asset will be
recorded in the amount of P30,000 to be depreciated over 10 years with annual
depreciation of P3,000.
27. (c) Fund raising, general, and administrative expenses are classified as supporting
services. Program services are for costs directly related to the organizational purposes.
28. (c) The cost of printing the annual report and the cost of an audit performed by a
CPA firm would be other general and administrative expenses. Since the merchandise is
being sent to encourage contributions, it would be a cost of fund-raising and reported
as such in the activity statement.
29. (c) Since the church will have the use of the principal after mal Lush dies, but will
have no right to the income during her life, the church will recognize the principal
balance of P100,000. The amount cannot be used currently, however, and will be
recorded as deferred support.
30. (b) The P150,000 paid to full-time staff will clearly be included in salary and wage
expenses for the organization. The P10,000 value of the services donated by the two
volunteers replacing the full-time secretary will also be recognized as salary and wage
expenses with an offsetting credit to support. The services volunteered for special
317
events by employees of local businesses would not be reported as salary and wage
expenses for reasons that there is no employer-employee relationship that. The total
amount to be reported will be P160.000.
31. (c) Pledges are recognized net of uncollectible amounts. Since total pledges are
P350,000, but 10% is expected to be uncollectible, pledges receivable will be recorded
in teh amount of P350,000, but an allowance for uncollectibility of 10% or P35,000 will
be established. The net amount of P315,000 will be reported as pledges receivable.
Less: Reclassified to unrestricted since it was already expended for equipment 35,000
P15,000
(a)
33. (d) The designation of unrestricted net assets by the board of Eddie Museum for the
building addition does not change the classification of the net assets which were
designated. the assets designated were unrestricted before the designation, and they
remain unrestricted after the designation by the board.
318
35. (d) According to FASB 117, donations of works of an for which the donor stipulated
a specified purpose and which are to be preserved and not be sold, represent
permanently restricted net assets. Since the museum's policy is to capitalize all
donations of art, Ms. Florendo donation should be reported as an increase in
permanently restricted net assets on the statement of activities.
37. (c) A temporary restriction is a donor-imposed restriction that permits the donee
organization to use up or expend the donated assets as specified, it is satisfied either
by the passage of time or by actions of the organizations involved. Accordingly, the
P5,000,000 contribution of Night Co. shares represents temporarily restricted net assets
until the shares are sold and the proceeds used to erect a public viewing building. The
P2,000,000 contribution of Night Co. shares represents permanently restricted net
assets because the shares are to be retained permanently.
38. (c) All expenses are presented as unrestricted on the statement of activities. 90. (c)
the restriction imposed is temporary (purchase new computer equipment purpose
restriction) not a permanent one.
39. (d) A statement of financial position for a nonprofit entity, like a library, should
report net assets according to whether the net assets are unrestricted, temporarily
restricted, or permanently restricted.
40. (a) According to FASB 1 17, the net assets of term endowments should be reported
as temporarily restricted, while the net assets of regular endowments should be
reported as permanently restricted. The net assets of term endowments are temporarily
restricted because the donor of a term endowment stipulates that the endowment last
319
only a specific number of years. Donors of regular endowments intend that these
endowments last indefinitely; hence, the net assets are permanently restricted.
320
Government Accounting – General Fund
Budget Process
321
GOVERNMENT ACCOUNTING – GENERAL FUND
322
OTHER SPECIAL TOPICS (BASIC KNOWLEDGE)
Accounting For Insurance Contracts by Insurers (PFRS 4)
323
OTHER SPECIAL TOPICS (BASIC KNOWLEDGE)
ACCOUNTING FOR BUILD, OPERATE AND TRANSFER (PFRIC 12)
324
OTHER SPECIAL TOPICS (BASIC KNOWLEDGE)
Effective Communications to Stakeholders
325
SYSTEM OF COST ACCUMULATION OR COSTING SYSTEM
1. D
2. A
3. C
4. B
5. B
6. D
7. D
8. E
9. D
10.A
11.D
12.A
13.A
14.E
15.C
16.B
17.A
18.D
326
1,600 x = P240 + P5,760
x= P3.75
19.C
20.D
21.B
22.C
x = 11,120
23.D
24.B
Budget allowance:
25.B
327
[(52,000 x P6) + P250,000]................................. P562,000
26.C
Budget allowance:
27.A
28.E
29.A
30.C
328
31.D
1,000 x = P3,000
x = P3
y= P2.80 - P3.00(30,000)
y= (P6,000) favorable
32.A
x = (P1,020) favorable
33.D
34.D
x = P2,000 unfavorable
35.C
36.D
37.D
329
38.A
39.D
40.B
1. E
2. C
3. A
4. D
5. A
6. D
7. B
8. A
9. E
10. A
11. A
330
12. D
13. C
14. A
15. C
16. B
17. B
18. C
19. D
20. E
21. A
22. D
23. C
24. E
25. D
26. B
27. A
28. D
29. C
331
30. B
31. C
32. B
33. B
34. B
35. A
36. E
37. A
38. D
39. D
40. A
1. Materials........................................................ 28,000
Accounts Payable....................................... 28,000
Materials.................................................... 21,000
4. Payroll........................................................... 31,000
Accrued Payroll.......................................... 31,000
332
Accrued Payroll.................................................... 31,000
Cash.......................................................... 31,000
Payroll....................................................... 31,000
9. Materials...................................................... 40,000
333
Accounts Payable......................................... 40,000
Materials...................................................... 31,000
Payroll......................................................... 48,000
334
Record direct materials used (P9,000+ 14,000)
16.Overhead 1,000
Raw Materials Inventory 1,000
18.Overhead 2,000
Factory Payroll 2,000
21.Accounts Receivable9,000
Sales 9,000
335
Record sale of Job 105 for P9,000 on account
23.Overhead 6,800
Accounts Payable 4,300
24.
Material inventory P4,000,000
26.
Inventories P2,800,000
27.
Inventories P400,000
28.
336
Work in process—PK03 P3,000,000
Cash P3,000,000
29.
Cash P1,000,000
30.
31.
32.
33.
Revenue P11,440,000
34.
337
Cost of goods sold P900,000
35.
36.
37.
Debit Credit
Inventory 100,000
338
Overhead cost pool 150,000
38.
Debit Credit
39.
Debit Credit
40.
Debit Credit
339
Direct material price 240
variance
1. C 12.B
2. A 13.A
3. A 14.C
4. C 15.C
5. B 16.C
6. D 17.A
7. D 18.B
8. C 19.A
9. B 20.B
10.D 21.B
11.D
22.D
340
Raw materials used P365
23.C
Total P730
24.C
Add cost of goods manufactured during the year (P680 + P50 - P65) 665
341
25.B 33.C
26.B 34.A
27.C 35.A
28.A 36.C
29.C 37.C
30.A 38.B
31.A 39.B
32.B 40.B
1. A
2. B
3. A
4. D
5. C
6. A
7. B
8. C
9. B
10.C
11.C
12.B
342
13.C
14.D
15.B
16.D
21. B
Explanation:
Calculation for
343
Assigned to:
Normal spoilage
22.
Explanation:
D) Calculation for
Assigned to:
344
(4,000 × P25; P40) P 100,000 P 160,000
Normal spoilage
23.
Explanation:
B) Calculation for
Assigned to:
Normal spoilage
345
(1,000 × P25; P40) 25,000 (4) 40,000
24. B 33. C
25. C 34. B
26. C 35. C
27. D 36. D
28. B 37. B
29. D 38. D
30. A 39. A
31. D 40. D
32. D
1. B
2. B
3. C
4. C
SUPPORTING CALCULATION: 60% ($2 + $3) = $3
5. D
6. E
346
7. E
8. D
9. E
10. E
11. A
12. C
SUPPORTING CALCULATION:
13. D
14. E
15. E
16. E
17. A
18. C
19. C
SUPPORTING CALCULATION:
20. B
347
because the contribution should be recognized at fair value.
21. B
SUPPORTING CALCULATION:
22. E
SUPPORTING CALCULATION:
$2.78
23. A
24. D
25. B
26. C
27. A
28. B
348
29. A
30. D
SUPPORTING CALCULATION:
31. E
SUPPORTING CALCULATION:
PROBLEM 1:
SOLUTION
349
Cost from preceding
PROBLEM 2
SOLUTION
350
(2) Work in process—Sanding Department (800 units x 3/4) 600
* All Sanding Department direct labor would be in all of these units or else they never
would have been transferred.
PROBLEM 3
SOLUTION
*Equivalent production:
Labor and factory overhead: 33,000 + 2,000 + (1/2 x 5,000) = 37,500 units
351
Factory overhead (5,000 units x 1/2 x $15.50) 38,750
$ 204,350
PROBLEM 4
SOLUTION
Materials 45,000
Payroll 75,000
352
PROBLEM 5
SOLUTION
Isogen Corporation
Planning Department
353
Received from Cutting Department 7,500
10,500
10,500
Beginning inventory:
Materials 7,800
Labor 3,200
% Unit Total
354
Cost Accounted for as Follows Units Complete Cost Cost
Transferred to Finishing
Work in process,
ending inventory:
* Total number of equivalent units required in the cost accounted for section
determined as follows:
Prior
PROBLEM 6
SOLUTION
355
Carlson Chemical Company
Blending Department
18,000
18,000
Beginning Inventory:
Materials 720
Labor 1,150
356
Cost added during period:
% Unit
Total
Transferred to Bottling
Work in process,
ending inventory:
* Total number of equivalent units required in the cost accounted for section
determined as follows:
357
Prior
PROBLEM 7
SOLUTION
Labor and
Factory
Materials Overhead
358
Units started and finished this period 11,000 11,000
PROBLEM 8
SOLUTION
Assembly Department
359
Received from Shaping Department 3,000
4,000
4,000
Beginning inventory:
Materials 7,700
Labor 3,500
% Unit Total
360
Cost Accounted for as Follows Units Complete Cost Cost
Cost to complete:
Work in process,
ending inventory:
* Number of equivalent units of cost added during the current period determined as
follows:
Prior
361
To complete beginning inventory 0 100 200 200
PROBLEM 9
SOLUTION:
Department 2
Cost of Production Report
For the Month of June, 19___
Quantity Schedule:
——- ——
362
Labor 56,100 6.80
——- ——-
——– ——-
363
Labor (5,000 × 25% ×$6.80) 8,500
——– 35,600
——–
Additional computations:
Equivalent production:
Journal Entries
1. D 11.A
2. B 12.C
3. C 13.C
4. D 14.C
5. C 15.D
6. D 16.C
7. A 17.B
8. D 18.A
9. D 19.C
10.D 20.C
364
21.C 31.C
22.D 32.C
23.B 33.A
24.C 34.A
25.D 35.D
26.D 36.A
27.D 37.A
28.D 38.B
29.A 39.D
30.D 40.C
1. D 11.B
2. A 12.D
3. B 13.D
4. B 14.D
5. B 15.A
6. B 16.C
7. A 17.B
8. D 18.C
9. B 19.A
10.D 20.B
365
21.A
22.A
Solution:
Units transferred Out 5,500
Less: Units in Beg. Inventory (300)
Units Started and Completed 5,200
23.C
Solution
Beg. Work in process 1500
Add: units started 9000
Less: Units transferred Out 7000
Ending Work in Process 3500
24.C
Solution:
Beginning Work in Process 5000
Add: Units started 54000
Less: Units transferred Out 47000
Ending Work in Process 12000
25.B
Solution:
The material is added at the beg. Of the process; therefore there are 22,000
equivalent units of material.
26. A
Solution:
366
Equivalent Units of Production 3,295
27. C
Solution:
Beg Work in Process 6,000
Add: Units started 24,000
Les: Units transferred Out 21,500
Ending Work in Process 8,500
28.D
Solution:
Material Costs
Beginning P23,400
29.B
Solution:
Equivalent Units:
Beginning Inventory (6,000/30%) P1,800
Started and Completed 15,500
Ending Inventory (8,500 * 10% 850
18,150 / 76,956 = 4.24 per unit
30. B
Solution:
Wreaths completed from BWIP 500
Wreaths started and completed 4400
4900
31. D
Solution:
Units transferred out 4,900
Cost per unit P 1.70
Total P 8,330
32.C
Solution:
367
The transferred-in cost component is the 8,000 units that were transferred
in.
33.A
Solution:
34.B
Solution:
35.A
Solution:
36.C
Solution:
368
Current costs Equivalent Units Cost/Equivalent Unit
37. C
Solution:
38.A
Solution:
Units started this period 32,000
Less: Ending Work in process 2,500
Units started and completed this period 29,500
39.A
Solution:
Materials are added at the beginning of the process. 32,000 units were
started in the current period; therefore there are 32,000 equivalent units for
materials.
40.B
41.B
Solution:
Equivalent Units:
Beginning Inventory (7000 * 100%) 7,000
Started and completed (29,500) 29,500
369
Ending Inventory (2500 * 25%) 625
43.D
Solution:
44.A
Solution:
Units started this period 120,000
Less: Ending Work in Process 8,200
Units started and completed this period 111,800
45. D
Solution:
Equivalent Inventory (10,000 * 100%) 10,000
Started and completed (111,800) 111,800
Ending Inventory (8,200 * 25%) 8,200
130,000 equivalent units
46. B
Solution:
Equivalent Units:
Beginning Inventory 0
Started and Completed 111,800
Ending Inventory (8,200 * 25%) 8,200
120,000 equivalent units
370
47.C
Solution:
48.D
Solution
49.C
Solution:
Material Costs:
Beginning P 24,500
Current Period P 75,600
P 100,000 / 130,000 = P .77 per unit
50.C
Solution:
Conversion Costs:
Beginning P 68,905
Current Period 130,053
198,958 / 128,60 = P 1.55 per unit.
371
51.D
Solution:
52.A
Solution:
Conversion Costs:
Beginning (Ignored)
Current Period 130,053
130,053 / 123,860 = P 1.05 per unit
53.A
Solution:
54.B
Solution:
55.C
372
Solution:
56.A
Solution:
FIFO
2,600
57.B
Solution:
FIFO
2180
58.A
373
Solution:
Weighted Average: Material A
Beginning P 14,270
Current Period P 40,000
P 54,270 / 2,700 = P20.10 per unit
59.C
Solution:
60.B
Solution:
61.D
Solution:
62.B
Solution:
374
Conversion Costs Equivalent Units Average Cost per EUP
(Current Period)
63.A
Solution:
64.C
Solution:
65.B
Solution:
375
Add: Unit started and completed 51,500 100% 51,500
66.A
Solution:
Weighted Average: materials
Beginning P 25,100
Current Period P 120,000
P 145,100 / 84,500 = P 1.72 per
unit
67.C
Solution:
Weighted Average Conversion:
Beginning P 50, 000
Current Period 300,000
P 350,000 / 78,100 = 4.48 per
unit
68.D
Solution:
No costs ae assigned to normal, continuous spoilage. Higher costs are
assigned to good units produced.
69.D
Solution:
376
16,000 P 1.75 P 28,000
P 71,680
70.D
Solution:
71.B
Solution:
377
Equivalent Units of Production 67,225
72.C
Solution:
FIFO: Materials
73.A
Solution:
FIFO: Conversion
74. A.
Solution:
75.A
378
Solution:
76.D
Solution:
77.C
Solution:
379
Absorbed spoilage is a period cost
78.B
Solution:
79.A.
380
SUPPORTING CALCULATION: 20,000 + .75(8,000) = 26,000
80.D
381
PROCESS COSTING SYSTEM
Accounting for normal and abnormal lost units
1. C 12.C
2. C 13.D
3. D 14.D
4. D 15.A
5. C 16.B
6. D 17.A
7. A 18.A
8. B 19.D
9. C 20.D
10.D 21.B
11.D
22.D
Solution:
Direct materials P 42,500
Direct labor 65,250
FOH 78,300
Direct materials – rework 13,550
Direct labor – rework 15,250
FOH – rework 18,300
Total cost 233,150
Cost per unit (233150/450) P518.11
23.C
Solution:
24.C
25.C
26.D
382
27.C
28.D
29.A
30.A
31.B
32.A
33.C
Solution:
Unit Material Cost = 14,000 + 58, 000 / 7,500 + 1,700 + 800 = 7.20
34.A
Solution:
Unit Conversion Cost = 20,000 + 104,500 / 7,500 + 800 + 60% (1,700)
= 13.3584 + 7.20
= 20.5584
35.A
Solution:
Units completed (7,500 * 20.5584) 154,188
Normal Spoilage 12,419
Total cost transferred to the next dept. 166,607
36.C
383
Solution:
Cost transferred to the lost account ( 50 * 20.5584) 828
37.D
38.A
39.A
40.A
Solution:
Unit material cost = 58,000 / 7,500 – 2,000 + 1,700 + 800
= P 7.250
41.C
Solution:
Unit Conversion Cost = 104,500 / 7,500 – 75% (2,000) + 60% (1,700) + 800
= 13.3631 + 7.250
= 20.6132
42.A
43.D
45.A
46.A
384
47.D
48.B
49.C
50.A
51.C
52.C
53.B
385
54.A
55.C
56.D
57.C 69.D
58.D 70.B
59.D 71.A
60.C 72.B
61.D 73.B
62.D 74.C
63.A 75.C
64.B 76.C
65.C 77.D
66.C 78.C
67.A 79.D
68.D 80.D
386
BACKFLUSH COSTING (JIT SYSTEM)
Cost Accumulation procedures- material, labor, overhead
1. D
2. A
3. C
4. C
5. D
6. B
2,000
= 4 days
500
7. D
8. D
9. A
10.B 30% x 1/2 x $300,000 = $45,000
11.A
12.C 50% x 20% x $75,000 = $7,500
13.D 50% x 20% x (10 x 100 x $75) = $7,500
14.B $25 x 5 x 250 = $31,250
15.B $25 x (50% x 100 x 10%) x (1/2 x 500) = $31,250
16.D
17.A
18.C
19.D
20.B
21.B
22.A
23.D
24.A
25.D
26.B
27.D
28.A
29.B
Carrying cost savings = 33% x reduction in average cost of WIP
387
= $19,800
30.B
Savings in cost of defects = $20 x reduction in the number of defective units
= $20 x 10 x 20
= $4,000
31.C
32.D
The average lead time will be 26 days, calculated as follows:
= 3/4 x 16 days
= 12 days
= 26 days
33.A
388
34.A
$250,000
= $50 per unit
5,000
35.B
389
BACKFLUSH COSTING (JIT SYSTEM)
Journal Entries
1. A
2. C
3. D
Supporting computation for 1-3
This is a summary entry for all receipts of raw materials during the period. As direct
materials are used, no entry is needed, because they remain a part of RIP.
This entry backflushes material cost from RIP to Finished Goods. This is a post
deduction. The calculation is:
$718,700
Conversion cost in RIP is adjusted from the $4,800 of August 1 to the $5,300
estimate at August 31. The offsetting entry is made to Cost of Goods Sold, where
all conversion costs were charged during August.
390
4. A
5. C
6. D
Supporting Computation for 4-6
This is a summary entry for all receipts of raw materials during the period. As direct
materials are used, no entry is needed because they remain a part of RIP.
This entry backflushes material cost from RIP to Cost of Goods Sold. This is a
postdeduction. The calculation is:
$194,600
Conversion cost in RIP is adjusted from the $900 of June 1 to the $1,100 estimate at
June 30. The offsetting entry is made to Cost of Goods Sold, where all conversion
costs were charged during June.
7. A
8. C
9. C
10.C
391
Supporting Computation for 7-10
$177,900
To backflush material cost from Finished Goods to Cost of Goods Sold. This is a
postdeduction. The calculation is:
$176,550
392
Finished Goods.................................................................. 450
Conversion cost in RIP is adjusted from $600 of May 1 to the $850 estimate at May
31. Conversion cost in Finished Goods is adjusted from the $2,000 at May 1 to the
$1,550 estimate at May 31.
11.A 20. A
12.A 21. A
13.C 22. A
14.C 23. B
15.C 24. C
16.B 25. D
17.B 26. A
18. B 27. C
19. A
28. D
Supporting Computation for 26-28
This is a summary entry for all receipts of raw materials during the period. As direct
materials are used, no entry is needed, because they remain a part of RIP.
This entry backflushes material cost from RIP to Finished Goods. This is a post
deduction. The calculation is:
$718,700
393
Material in August 31 RIP, per physical count...................... 41,900
Conversion cost in RIP is adjusted from the $4,800 of August 1 to the $5,300
estimate at August 31. The offsetting entry is made to Cost of Goods Sold, where
all conversion costs were charged during August.
29.A
30.C
31.D
Supporting Computation for 29-31
This is a summary entry for all receipts of raw materials during the period. As direct
materials are used, no entry is needed because they remain a part of RIP.
This entry backflushes material cost from RIP to Cost of Goods Sold. This is a
postdeduction. The calculation is:
$194,600
394
Cost of Goods Sold........................................................ 200
Conversion cost in RIP is adjusted from the $900 of June 1 to the $1,100 estimate at
June 30. The offsetting entry is made to Cost of Goods Sold, where all conversion
costs were charged during June.
32.A
33.C
34.C
35.C
Supporting Computation for 32-35
$177,900
395
To backflush material cost from Finished Goods to Cost of Goods Sold. This is a
postdeduction. The calculation is:
$176,550
Conversion cost in RIP is adjusted from $600 of May 1 to the $850 estimate at May
31. Conversion cost in Finished Goods is adjusted from the $2,000 at May 1 to the
$1,550 estimate at May 31.
36.A
37.A
38.A
39.A
40.B
396
SERVICE COST ALLOCATION
Direct Method
1. C
2. A
3. C
4. D
5. A
6. A
7. C
8. B
9. D
10.C
11.A
12.D
13.B
14.A
15.B $100,000 × 90,000 / (90,000 + 100,000) = $47,368
16.D $150,000 × 2,000 / (2,000 + 2,500) = $66,667
17.A-The direct method does not allocate support department’s cost to other
support departments.
18.A
19.B
20.C
Computation for 18-20
Direct Method
Fabricating Assembly Finishing Total
No. of employees 30 40 20 90
Service percentage 33.33% 44.44% 22.22% 100.00%
Square feet 10,000 28,000 15,000 53,000
Service percentage 18.87% 52.83% 28.30% 100.00%
Admin. cost allocation*$16,666.67 $22,222.22 $11,111.11 $50,000.00
Janitorial cost all0.**5,660.38 15,849.06 8,490.57 30,000.00
Direct overhead cost 40,000.00 50,000.00 25,000.00
Total overhead cost $62,327.05 $88,071.28 $44,601.68
Div. by direct labor hrs.÷ 5,000 ÷ 6,000 ÷ 2,000
Dep. OH rate per DLH$ 12.47 $ 14.68 $ 22.30
**Departmental employee % × $50,000
**Departmental square feet % × $30,000
397
21.A
22.D
23.C
24.B
25.B
26.A
27.D
28.C
29.B
30.A
31.A
32.B
33.C
34.A
35.A
36.A
37.A
38.A
39.A
40.C
398
Service Cost Allocation
Step Down
1. C 18.D
2. C 19.B
3. A 20.A
4. A 21.A
5. A 22.A
6. D 23.C
7. C 24.B
8. A 25.A
9. C 26.A
10.D 27.C
11.B 28.A
12.A 29.A
13.A 30.B
14.A 31.D
15.D 32.D
16.C 33.A
17.B
34.A
35.B $275,000 × 125 / (5 + 125 + 100) = $149,457
36.D $250,000 + ($275,000 × 5) / (5 + 125 + 100) × 1,500 / (3,500 + 1,500) =
$255,978 × 1,500 / 5,000 = $76,793
37.B -The Repair Department performs allocation first. Thus, the Tool Department
will receive cost allocation of ($35,000 × 1/40) = $875.
38.A
39.B
40.C
399
Service Proportion Table
Admin. Janitorial Fabricating Assembly Finishing
Total
No. of employees................. 10 30 40 20 100
Service percentage............ 10.00% 30.00% 40.00% 20.00% 100.00%
Square feet.......................... 2,000 10,000 28,000 15,000 55,000
Service percentage............ 3.64% 18.18% 50.91% 27.27% 100.00%
Sequential Cost Allocation
Admin. Janitorial Fabricating Assembly Finishing
Total
Direct overhead cost.......... $ 50,000) $30,000) $40,000 $50,000 $25,000
First step:
Allocate admin. costs............ $(50,000)$ 5,000) $15,000 $20,000 $10,000
Second step:
Allocate janitorial costs
Determine allocation percentages:
Square feet................... 10,000 28,000 15,000 53,000
Allocation percentage..... 18.87% 52.83% 28.30% 100.00%
Janitorial cost allocation.... $(35,000) $6,604 $18,491 $9,906 $35,000
Total overhead costs............ $0) $0) $61,604 $88,491 $44,906
Divided by direct labor hours ÷ 5,000 ÷ 6,000 ÷ 2,000
Departmental OH rate per DLH $ 12.32 $ 14.75$ 22.45
400
1. D
2. D
3. D
4. C
5. B
6. D
7. D
8. D
9. C
10.D
11.C
12.A
13.B
14.D
15.B
16.C
17.B
18.D
19.A
20.C
21.D
22.C
23.C
24.D
25.D
26.D
27.A
28.B
29.B
30.A
31.B
32.C
Supporting Computation for 30-31
Service Proportion Table
Admin. JanitorialFabricatingAssemblyFinishing
Total
No. of employees................. 10 30 40 20 100
Service percentage............ 10.00% 30.00% 40.00% 20.00%
100.00%
Square feet.......................... 2,000 10,000 28,000 15,000 55,000
Service percentage............ 3.64% 18.18% 50.91% 27.27%
100.00%
Simultaneous equations can be created based on the service proportion table above.
A = $50,000 +.0364J J = $30,000 + 0.1A
where A = Administration Department total
costs
J = Janitorial Department total costs
A = $50,000 + .0364 × ($30,000 + 0.1A)
A = $50,000 + $1,092 + 0.00364A J = $30,000 + (0.1 × $51,279)
0.99636A = $51,092 J = $30,000 + $5,128
A = $51,279 J = $35,128
Solution:
1. C
The traditional approach lumps all the overhead costs into one pool and allocates
it based on one activity.
402
Allocation Rate:($96,000 + $180,000 + $34,000)/[(700 x 2) x (2,000 x
3.3) = $38.75
Amount allocated to each 'Fred': $38.75 x 2 hours per unit = $77.50
2. A
Setups: $96,000 x 36/48 = $ 72,000
3. B
4. A
$250,000/5,000 = $50 per DLH
5. A
$50 per DLH x 2 DLH per Bert = $100
6. C
7. A
8. B
9. C
Overhead rate = [$1,225,000 + $175,000]/175,000 = $8 per direct labor hour
403
10.A
Overhead rate = $261,780/3,000DLH = $ 87.26/DLH
Allocated overhead:
Product A: $87.26 * 1DLH= $87.26
Product B: $87.26 * 2DLH= $174.52
11. B
Activity center
rates:
Setups $27,140/46 = $590 / setup
Allocated overhead:
Product A Product B
91,480 170,300
Overhead per
$ 182.96 $ 136.24
unit
12.A
$600/400 x 1.5 hours = $2.25
13.A
[$100,000/500 x 2] = $400
14.D
[$24,000/1,200] x 10 = $200
15.C
[$1,000,000/500,000] = $2.00 per machine hour
16.A
404
Direct material $30,000
Security 5,000
Total $59,500
17. A
$50 x 12 = $600
Ordering: $22,500/450 = $50 per PO
Total $1,928
18.B
19.B
Total cost allocated = $10.20 Total cost allocated = $10.00
405
20.C
21.A
25.C
Setups: $16,500/250 = $66/ setup $66 x 24 setups = $1,584
Total = $14,184
26.D
$ 28,000 = $56 per requisition x 150 requsitions = $ 8,400
500
12,000
407
$ 72,000 = $360 per service call x 180 calls = 64,800
27.C
TOTAL $41,930
28.A
[$72,000 + $160,000 + $80,000]/10,400 DLH = $30.00 per DLH
29. A
$30.00 per DLH x 3 hours = $90
30.D
Machine setups $72,000/192 setups = $375 per setup x 144 setups =
31.D
Setup: $50,000 x 4/40 = $5,000
408
Total $20,600
32.C
From Dept. 1: 1,500/(1,000 + 1,500) x $400,000 = $240,000
TOTAL $615,000
33. B
34. A
35. B
36. D
Direct material $30,000
Security 5,000
Total $59,500
37.B
$104,500 x 20,000/100,000 = $20,900
38.A
39. B
Machine Setup $60,800/320 x 140 = $26,600
409
Per unit cost = $93,600/20,000 buckets = $4.68
40.C
410
ACTIVITY-BASED COSTING SYSTEM
1. E
2. C
Activity 1 (P20,000 x 100/500) P 4,000
Activity 2 (P37,000 x 800/1,000) 29,600
Activity 3 (P91,200 x 800/3,800) 19,200
Total allocated cost P52,800
÷ number of units 8,000
Cost per unit P 6.60
3. A
Activity costs, Patient 2:
Room and meals (3 x P150) P 450
Radiology (2 x P95) 190
Pharmacy (1 x P28) 28
Chemistry lab (2 x P85) 170
Operating room (1 x P550) 550
Total P1,388
4. D
5. B
Prime costs P 900,000
Applied overhead ( P600,000 / 75,000 DLH x 75,700) 605,600
Total cost P1,505,600
÷ Units produced 100,000
Unit cost P 15.06
6. A P 15.06
7. D
8. C
9. A
10.E
11.E
411
12.D
High $ 100,000 5,000
Low 80,000 2,500
Difference $ 20,000 2,500
Variable rate = $20,000 2,500 = $8.00/direct labor hour
13.A
14.B
15.B
16.C
17.D
18.A
19.A
20.E
21.A
22.D
23.B
24.C
25.C
26.A
The existing system allocated 3,000/30,000 = 10% of all overhead to Product A
last year; but A accounted for only 25/500 = 5% of batch-level activity. So, with
respect to batch-level costs only, the existing system overstated A's cost
last year by a total of:
35.D 38.C
36.D 39.A
37.A 40.C
413
Methods of allocating joint cost to products
1. C. -If Sonimad were to process the MSB further, they would incur an
additional cost of P100,000. They would also be able to sell the 60,000 units
for P3 more per unit. This is an additional P180,000 in revenue. When
reduced by the additional costs, Sonimad would increase their contribution
by P80,000 if they further processed the MSB. By not processing the MSB
further, their contribution would be P80,000 less.
2. A. -In order to allocate the joint costs using the physical-quantity basis, we
need to determine the total physical quantity that was produced. There were
180,000 gallons of LS and 120,000 gallons of SS. In total there were 300,000
gallons, of which SS comprised 40%. Therefore, SS should receive 40% of
the P420,000 in joint costs. This is P168,000.
3. D. -Joint products are identifiable at the split-off point and they have
significant sales value.
4. B. In order to allocate the costs using the relative sales value method, we
need to know the total relative sales value. There are 60,000 units of MSB
that will be sold at P2 each for a total value of P120,000. There are 90,000
units of CBL that will be sold for P4 each, for a total value of P360,000. The
total sales value of both products is P480,000, and MSB represents 25% of
this total sales value. The joint costs are P300,000 and MSB should be
allocated 25%, or P75,000, of the joint costs.
5. B.-In order to allocate joint costs based on the relative sales value of the
output, we need calculate the sales value of the output for each product. For
Two Oil it is P6,000,000 (300,000 barrels produced × P20 each). For Six Oil it
is P7,200,000 (240,000 barrels produced × P30 each). For Distillates it is
P1,800,000 (120,000 barrels produced × P15 each). In total, this is
P15,000,000, of which Two Oil is 40%. Therefore, Two Oil will be allocated
40% of the P10,000,000 in joint costs, or P4,000,000.
6. B.- In order to calculate the total cost of Product S, we need to know how
much of the joint costs will be allocated to Product S. In order to allocate
414
joint costs based on the sales value at the split off point, we need to
determine the sales value at the split off point for each item. For R it is
P250,000 (2,500 units produced × P100). For S it is P400,000 (5,000 units ×
P80). For T it is P150,000 (7,500 × P20). In total this is P800,000, of which
Product S is 50%.
Therefore, 50% of the joint costs should be allocated to Product S. Joint costs
were P720,000, so Product S receives P360,000. For the company it is more
beneficial to process product S further because it gains greater incremental
revenue after further production. Thus, we need to add the P150,000 of
processing costs after the split off point to determine the total cost of Product S
as P510,000.
7. C
MV of by-product Zest 5
Operating profit 1
10. D-Market value in this situation requires the calculation of the net realizable
415
value of each product because the joint products cannot be sold at the split-
off point.
The net realizable value per unit of product A is P48 (selling price P60 less P12
further processing cost) and of product B is P88 (selling price P104 less P16
further processing cost). The proportion of the process costs apportioned to
product B is [(3,000 × 88)/(2,500 × 48) + (3,000 × 88)].
Thus, P192,000 × 264/384 = P132,000.
11.B-A product should be sold at the split-off point if there is not any
incremental profit from processing the product further. As long as the process
as a whole is profitable, it is irrelevant if an individual product is not
profitable. It has to be assumed, in this example, that the process as a whole
is profitable.
Product A Product B
15. D
A 30,000*15,000/(15,000+60,000)
416
16. C
B 30,000 – 6,000
17.C
Product Sales value at SO point Joint Costs
V-1 P420,000@60% P252,000
18.B
Product Joint Cost Further Total Costs
Processing
Costs
19.B
Product Joint Cost Further Total Costs
Processing
Costs
20.B
21.C
Product MV at Split-off Joint costs Further Process Total Cost
point Cost
417
22.C
MV at Joint
SoPt. X %* Costs
P200,000 P120,000
*P120,000/P200,000= 60%
23.C
Sales of product E if
process further P40,000
Further
processing costs 6,000 24,000
418
24.C
Sales of product W if process
further P78,000
25. D
Total Joint Costs = P375,000-P50,000=P325,000
419
NRV
26.B
Lead (20,000-8,000) P12,000
Manganese (30,000-
6,000) 24,000
Total P75,000
27.C
MV at SoPt. X %* Joint Costs
Q ?
R P15,000
P100,000 P60,000
*P60,000/P100,000=
**P36,000/60%=P60,0
00
28. Market value in this situation requires the calculation of the net realizable
value of each product because the joint products cannot be sold at the split-
off point.
The net realizable value per unit of product A is P48 (selling price P60 less
P12 further processing cost) and of product B is P88 (selling price P104 less
P16 further processing cost). The proportion of the process costs apportioned
to product A is [(2,500 × 48)/(3,000 × 88) + (2,500 × 48)].
Thus, P192,000 × 120/384 = P60,000.
420
29.C
Sales of product W if process
further P538,000
30.A
Sales of product W if process
further P320,000
31.A
A 50,000*20,000/(20,000+80,000)
32. C
B 50,000 – 6,000
33.D
421
34.A
35.C
36.C
MV at Joint
SoPt. X %* Costs
P200,000 P120,000
*P120,000/P200,000= 60%
37.D
MV at Joint
SoPt. X %* Costs
P200,000 P120,000
*P120,000/P200,000= 60%
422
**E's share in joint costs would be : P30,000X60%=P18,000
38.B
39.B
40.C
MV at Joint
SoPt. X %* Costs
75.5 P113,25
X P150,000 % 0
75.5
Y 350,000 % 264,250
75.5
Z 400,000 % 302,000
P679,50
P900,000 0
423
Methods of allocating joint cost to products
1. C. -Usually the sales revenue from the sale of a by-product is accounted for as
a reduction of the common costs that are allocated to the other products.
2. A-The NRV is calculated as sales price minus costs required to complete and
dispose of the item. In order to allocate joint costs using NRV we first need to
calculate the NRV for each of the two products. There are
60,000 units of MSB that will be sold for P5 each after further processing, or
P300,000. There are also P100,000 of additional costs that will be incurred, so
the NRV of MSB is P200,000. There are only 80,000 good units of CBL produced
because 10,000 units are damaged in further processing. These units will be sold
for P10 each, or a total of P800,000. However, there are also additional costs of
P200,000, making the NRV of CBL only P600,000. In total there is an NRV of
P800,000, of which CBL makes up 75%. Therefore, CBL should receive 75% of
the joint costs, or P225,000. The question is the total cost that would be
assigned to a CBL unit. In addition to the joint costs, we also need to include the
additional processing costs of P200,000. In
total, then, there are P425,000 of costs that need to be allocated to the 80,000
units of CBL. This is P5.3125 per unit.
3. C.- The NRV is calculated as sales price minus costs required to complete and
dispose of the item. In order to allocate joint costs using NRV we first need to
calculate the NRV for each of the two products. This process is easier in this
question because there are no further costs to worry about so their NRV is
simply equal to their selling price. There are 180,000 gallons of LS that will sell
for P2.40 per gallon, or P432,000 total. There are 120,000 gallons of SL that will
sell for P3.90 per gallon, P468,000 in total. Together, these two products have a
NRV of P900,000, of which LS represents 48%. Therefore, LS should be allocated
424
48% of the joint costs. The joint costs were P420,000 and LS's share of this is
P201,600.
4. D.-The joint costs to be allocated total P5,000 (P2,000 to purchase Duo and
P3,000 to process it into Big and Mini). The net realizable value for Giant is (600
× P17) minus the additional processing costs of P1,000, which equals P9,200.
The sales price for Mini will be used as its net realizable value in this allocation,
since it will not be processed further. That is (200 × P4), or P800. The total of
the two values is P10,000 (P9,200 + P800), and Giant's NRV of P9,200
represents 92% of that. Therefore, 92% of the joint cost of P5,000, or P4,600,
will be allocated to Giant.
However, the question asks for the total cost of producing Giant, not just the
amount of the joint cost allocated to Giant. In addition to the joint cost of
P4,600, there will be P1,000 of additional processing costs incurred by Giant.
Therefore, the total cost of producing Giant is P4,600 + P1,000, or P5,600.
5. A. - Net realizable value (NRV) is calculated as the selling price minus future
costs to complete and dispose. For Alfa, the NRV is P2 per pound, or P20,000 in
total. For Betters, the NRV is P8 per pound, or P40,000 in total. Together, these
two products have P60,000 of NRV. Of this, 33% is Alfa and 67% is Betters.
Therefore, Betters will get 2/3 of the joint allocable costs. The joint costs to
allocate are the P93,000 and because they do not inventory the by-product, all
P93,000 of the joint costs need to be allocated. Betters is to receive 2/3 of this,
or P62,000.
6. A.- The NRV of an item is calculated as the selling price minus the costs to
complete and dispose. In order to allocate joint costs based on NRV, we need to
calculate the total NRV that all of the joint products have at the split off point. In
this question there are no further processing costs, so we will use the sales price
as the NRV. One pound of cheese has a sales price of P2 and there were 450
pounds, so this is P900 of NRV. One pound of whey has a sales price of P.80 and
there will be only 375 pounds of whey that will be able to be sold, so this is P300
(the whey that will not be sold will bring no NRV to the company since the sales
price is P0). In total there are P1,200 of NRV, and the cheese accounts for 75%
of it 900 ÷ 1,200). This means that the cheese will get 75% of the common
costs. Now we need to calculate the common costs. They are made up of P200
of milk, P400 of labor and P400 of overhead. This is P1,000 and 75% of it is
P750. The other P250 will go to the whey.
7. C.- The NRV is calculated as sales price minus costs required to complete and
dispose of the item. In order to allocate joint costs using NRV we first need to
425
calculate the NRV for each of the two products. We can do this on a per unit
basis as follows: The selling price of LS is P2.40 per gallon, but there are
additional processing costs of P1.40 per gallon. This leaves an NRV of P1 per
gallon of LS.
Since there are 180,000 of LS, the NRV for LS is P180,000. Doing the same for
SS, we get an NRV of P3 per gallon (P3.90 selling price P.90 additional costs)
and with 120,000 gallons this is a total NRV of P360,000. In total there is
P540,000 of NRV, and SS makes up 2/3 of the total NRV. Therefore, SS should
receive 2/3 of the P420,000 joint costs, or P280,000.
8. D.- Because both products can be sold at the split off point, we must use the
relative sales values at the split-off point in order to allocate the joint costs, even
though one product will be processed further. Using the relative sales value
method, we will allocate the P10,000 of joint costs to the different products.
Since product X has a sales value at split off of P12,000 and Y has a sales value
at split off of P8,000, the total sales value is P20,000. 60% of this is from
product X, so product X will receive 60%, or P6,000, of the joint costs.
9. C.- Net realizable value (NRV) is calculated as the selling price minus future costs
to complete and dispose. For Alfa, the NRV is P2 per pound, or P20,000 in total.
For Betters, the NRV is P8 per pound, or P40,000 in total. Together, these two
products have P60,000 of NRV. Of this, 33% is Alfa and 67% is Betters.
Therefore, Alfa will get 1/3 of the joint allocable costs. The joint costs to allocate
are the P93,000 in joint costs reduced by the P3,000 of revenue that will be
received from the sale of the by-product. (The process of inventorying the by-
product requires that the P3 per unit sales value is debited to inventory and the
corresponding credit is a reduction of the production costs.) So, P90,000 needs
to be allocated, and 1/3 of this is P30,000.
10.B. - The NRV of an item is calculated as the selling price minus the costs to
complete and dispose. In order to allocate joint costs based on NRV, we need to
calculate the total NRV that all of the joint products have at the split off point.
We will make these calculations based on one ton of raw materials. We can do
this because the answer is in %, not in the dollar amount. F1 has an NRV of P2
per unit and there are 5 units, so this is P10 of NRV.
F2 has an NRV of P5 per unit and there are 2 units, so this is P10 of NRV. F3 has
an NRV of P10 per unit and there are three units, so this is P30 of NRV. In total
there is P50 of NRV and the NRV of F1 is 20% of this amount.
426
11.
(P100,000/140,000)*P60,000
P42,857
12.
Apportioned Joint Cost for Chlorine:
(P100,000/140,000)*P80,000
P57,143
13. A.
Net
realizable value method
14.D.
15.B.
16.A
NRV P20,000
17.B
18.B
19.C
20.D
A 30,000*11,000/(11,000+58,000)
B 30,000 – 4,783
427
21.
22.
23.
24.
25. B
Product Unit Produced Market Value Market Value – Proc. Costs= Hy. MV
P400,000
P400,000 P120,000
*P120,000/P400,000
428
26. D - P120,000-66,000=P54,000
27. A
50,000 P120,000
P500,000
P500,000 P200,000
429
*P200,000 / P500,000 = 40%
28. C
29. A
30. C P50,00*60%=P30,000
32. D. - Because both products can be sold at the splitoff point, we must use the
relative sales values at the splitoff point in order to allocate the joint costs, even though
one product will be processed further. Using the relative sales value method, we will
allocate the P10,000 of joint costs to the different products. Since product X has a sales
value at splitoff of P12,000 and Y has a sales value at splitoff of P8,000, the total sales
value is P20,000. 60% of this is from product X, so product X will receive 60%, or
P6,000, of the joint costs. (Question 29 - CIA 1185 IV-11 - Joint Products and
Byproducts)
430
37. A. Net realizable value
39.C
40.D
A 30,000*11,000/(11,000+58,000)
B 30,000 – 4,783
1. D. -Using physical quantity, there are a total of 15,000 pounds of Alfa and
Betters. Of this, 2/3 is Alfa and 1/3 is Betters. Therefore, Alfa will receive 2/3 of
the joint costs. The joint costs are P93,000, but because the by-product is
being inventoried, this needs to be reduced by the P3,000 revenue from selling
by-product. Therefore, there is a total of P90,000 that needs to be allocated
and 2/3 of this is P60,000.
2. C.- There are a total of P10,000,000 in joint costs that need to be allocated.
Under the physical output basis we first need to calculate the total physical
output. There were 660,000 barrels produced. Six Oil represented 240,000
barrels, or 36.36% of the total output. Therefore, Six Oil should get 36.36%,
or P3,636,363 of the joint costs. Note: if you used only 36%, you get a close,
but not exact answer.
3. A.- In this question the first thing we need to do is to reduce the joint costs that
need to be allocated by the sales value of the by-product. The sales value of the
by-product is P120,000 and this will reduce the costs to allocate to P2,400,000.
These costs are to be allocated based upon the physical volume of the two
products. Product 1 has a volume of 90,000 pounds and Product 2 has a volume
of 150,000 pounds. In total this is 240,000 pounds and since the 2nd product is
150,000 pounds, the allocation is as follows: P2,400,000 × (150,000 / 240,000)
= P1,500,000.
4. A.- In order to allocate the joint costs using the physical-quantity basis, we
need to determine the total physical quantity that was produced. There
were 180,000 gallons of LS and 120,000 gallons of SS. In total there were
300,000 gallons, of which LS comprised 60%. Therefore, LS should receive
431
60% of the P420,000 in joint costs. This is P252,000. Note: Because this
used the physical quantity as the allocation basis, the information about
further processing costs was not necessary since that is used in calculating
the Net Realizable Value.
5. A. -In order to allocate the costs using the physical quantity method, we need to
know the total physical quantity. There are 60,000 units of MSB and 90,000 units
of CBL. There are 150,000 total units and CBL represents 60% of the total units.
The joint costs are P300,000 and CBL should be allocated 60%, or P180,000, of
the joint costs.
6. C.- When physical quantities are used to allocate joint costs, it is possible that
the costs that are allocated to the units do not have a corresponding relationship
to the value.
8. A
Coke
Tar
9. B
Output in tonnes 1,200
Wastage in tonnes 50
432
Joint Cost @ P15 per
tonne 18,750
10.C
Sulphate of
Ammonia
Output in tonnes 52
Wastage in tonnes 2
11.A
Benzol
Output in tonnes 48
Wastage in tonnes 2
12.A
433
Coke 200/4,800*3,500=146
13.A
Tar 200/4,800*1,200=50
14.B
Sulphate of ammonia 200/4,800*52=2
15.B
Benzol 200/4,800*48=2
16.C
17.D
18. A
19.A
20. D
21. A
22.B
434
Apportioned joint cost:80,000/200 000*2,500,000=P1,000,000
23.D
24.A
25. C
26.D
27.A
28. C
29. B
30. C
31. D
32. A
33. B
435
34.A
35. A
36. B
37. C
38.D
39.A
40. B
436
Methods of allocating joint cost to products
Market (sales) value method
Weighted average method
1. B. -The NRV of an item is calculated as the selling price minus the costs to complete
and dispose. In order to allocate joint costs based on NRV, we need to calculate the
total NRV that all of the joint products have at the split off point. We will make these
calculations based on one ton of raw materials. We can do this because the answer
is in %, not in the dollar amount. F1 has an NRV of P2 per unit and there are 5
units, so this is P10 of NRV. F2 has an NRV of P5 per unit and there are 2 units, so
this is P10 of NRV. F3 has an NRV of P10 per unit and there are three units, so this
is P30 of NRV. In total there is P50 of NRV and the NRV of F1 is 20% of this
amount.
2. D-The total calories of Alfa are 44,000,000 and the total calories of Betters are
56,000,000. In total, this is 100,000,000 calories, of which Better is 56%. The total
joint costs to allocate are P90,000. This is made up of the P93,000 in joint costs
reduced by the inventoried sales value of the by-product. Betters is to receive 56%
of this P90,000, or P50,400.
3. C. -The market value of Alfa is P40,000 and the gross market value of Betters is
P50,000. In total, this is P90,000. If the sales value of the by-product is inventoried,
the total joint costs to allocate are P90,000. Of this, P40,000 would be allocated to
Alfa.
437
4. D.-All of the above methods are acceptable approaches to accounting for by-
products.
438
5. C.- The primary reason for allocating joint costs is for inventory valuation for financial
reporting.
10.A.
Units Ave. Unit Joint Further Proc. Total
Product produced Cost Costs Cost cost
*P90,000/12,000=P7.
A 4,000 P7.5 P30,000 50,000** P80,000
5
B 12,000 P7.5 90,000
**Ultimate market value or final sales: P10,000
P25X4,000 16,000 120,000
P50,000
1. D
16,000 120,000
*P90,000/12,000=P7.
5
P50,000
11.C
Average Cost per unit :P300,000/50,000=P6
12.D
Average Cost per unit :P200,000/400,000=P5
13.A
Average Cost per unit :P100,000/10,000=P10
14.A
Average Cost per unit :P100,000/30,000=P3.33
15.B
Average Cost per unit :P85,000/40,000=P2.13
16.A
Average Cost per unit :P100,000/30,000=3.33
17.C
Allocated joint cost: 85/215*50,000=P19,767
18.D
Allocated joint cost: 130/215*50,000=P30,232
22.C
5,000*11,200=56,000,000
xP0.0009
P50,400
23.B
10,000*4,400=44,000,000
X P0.0009
P39,600
440
24.C
Apportioned cost:50/200*70,000=P17,500
25.D
Apportioned cost:150/200*70,000=P52,500
26.D
Unit cost:175,000/50,000=P3.5
27.D
Unit cost:100,000/40,000=P2.5
28.A
Unit cost:250,000/80,000=P3.13
29.C
Unit cost:75,000/25,000=P3.00
30.C
Total cost: 3,000/10,000*250,000+50,000=P125,000
31.C
Total cost: 4,000/10,000*250,000+75,000=P175,000
32.C
Total cost:3,000/10,000*250,000+125,000=P200,000
33.A
Physical measure of units of output
34.C
Revenues of several product
35.A.
36.A. Marketing costs at point of sale
37.A.
38.B
39.C
441
METHODS OF ALLOCATING JOINT COSTS TO BY-PRODUCTS
1. C.- To determine when a product should be sold, we need to look at the costs to
manufacture further and the incremental revenue that will be received by
manufacturing further. The costs incurred after the split-off point are the costs of
further manufacture and are therefore relevant to this decision. (Question 19 -
CMA 1292 3-4 - Joint Products and Byproducts)
2. A. By-products
3. D
If the by-product is accounted for at time of production, by product inventory is
recorded at its selling price (or NRV in this case, given separable by-product
costs) because by-products usually do not receive an allocation of joint costs.
Thus, the by-product’s cost of sales is zero. Assuming sales of the by-product
reduced joint costs, the cost of sales of the gasoline was P100,000 (P120,000
cost to split-off –P30,000 sales of the by-product +P25,000 additional by-product
costs – P15,000 EI). Refer to No. 38 for further discussion.
Answer (A) is incorrect because the cost of sales of the by-product is P0. The
cost of sales of the gasoline equals P105,000 if the NRV of the by-product is
treated as other revenue. Answer (B) is incorrect because P115,000 ignores the
P15,000 ending inventory. Answer (C) is incorrect because allocating joint costs
to by-product based on relative sales value is not cost effective and by-product
cost of sales should be P0.
4. B
The difference between treating the by-product “May” as a joint product against
a by-product would be under the by-product treatment, the selling cost is netted
against “May’s” selling price, thus, reducing gross profit whereas under joint
product accounting, the selling costs would be deducted below the gross profit
like a selling expense.
Thus, if the change to joint-product accounting were made, gross profit would
increase and (b) therefore is correct.
5. A
The gross margin equals sales minus cost of sales. Before the change, the net
amount was deducted from the cost of sales (i.e., it increased the gross margin).
442
After the change, the net amount is added to regular sales with no additional
increase in cost of goods sold. Hence, the gross margin will be the same.
Answers (b), (c) and (d) are incorrect because the change in accounting method
has no effect on gross margin.
6. C
Sales revenue minus cost of goods sold is gross margin. If the net revenue from
the by-product is recorded as other income rather than being deducted from cost
of goods sold, the gross margin will decrease by P3,000 [1,000 x (P4 sales – P1
CGS)]
Answer (a) is incorrect because gross margin will be affected. Answer (b) is
incorrect because the gross margin will decrease by P3,000. Answer (d) is
incorrect because P4,000 is the gross amount of Zafa sales.
7. C
The NRV is selling price minus cost to complete and cost to dispose. The selling
price of Zafa is P4 and the selling costs are P1. Given no completion or additional
processing costs, unit net realizable value is P3.
Answer (a) is incorrect because P1 is the selling cost. Answer (b) is incorrect
because the unit net realizable value is P3. Answer (d) is incorrect because P4 is
the selling price of Zafa.
8. B
If the 1,500 units of the by-product are recognized at the time of production, net
income must increase by P4,500 (P3 unit NRV x 1,500). Ending inventory of Zafa
reduces cost of sales and by-product revenue either decreases costs or increases
other income.
Answer (a) is incorrect because P6,000 is the potential gross revenue from Zafa.
Answer (c) is incorrect because P3,000 is the net revenue from sales of 1,000
units. Answer (d) is incorrect because P1,500 equals 1,500 units times the P1
unit selling cost.
443
9. A
Because net realizable value is selling price minus completion and disposal cost,
there is no profit upon sale. The sale of 500 units of Zafa with an inventory value
of P3 per unit will produce no profit (P4 unit selling price – P3 inventory cost –
P1 selling cost =P0).
Answer (b) is incorrect because P500 equals 500 units times the P1 selling cost.
Answer (c) is incorrect because P1,000 is the selling cost of 1,000 units. Answer
(d) is incorrect because P1,500 is the P3 inventory cost multiplied by the 500
units sold.
10.A
Total Production Cost P99,538
*Cost per Kilo =------------------------------- = --------------- = P78.50 per kilo
Total kilos Produced 1,268
11.A. Scrap
12.B. waste
13.C. By-products
444
17.C. Approximated Net realizable value at split-off allocation
23.A.
Dr Work in Process Inventory
Cr Various accounts
28.D
Apportioned cost: 150,000/200,000*70,000= P52,500
29.D
Unit Cost: 175,000/50,000=P3.5
30.D
Unit cost: 100,000/40,000=P2.5
31.A
Unit Cost: 250,000/80,000=P3.13
32.C
Unit cost: 75,000/25,000=P3.00
33.B
When two or more separate products are produced by a common manufacturing
process from a common input, the outputs from the process are called joint
products. The common costs of two or more joint products with significant
values are generally allocated to the joint products based upon the products’ net
445
realizable values at the point they became separate products.
34. C. -If Sonimad were to process the MSB further, they would incur an additional
cost of P100,000. They would also be able to sell the 60,000 units for P3 more
per unit. This is an additional P180,000 in revenue. When reduced by the
additional costs, Sonimad would increase their contribution by P80,000 if they
further processed the MSB. By not processing the MSB further, their
contribution would be P80,000 less.
35.D. -Joint products are identifiable at the split-off point and they have significant
sales value.
36.B
A product should be sold at the split-off point if there is not any incremental
profit from processing the product further. As long as the process as a whole is
profitable, it is irrelevant if an individual product is not profitable. It has to be
assumed, in this example, that the process as a whole is profitable.
Product A Product B
446
ACCOUNTING FOR JOINT AND BY-PRODUCTS
TREATMENT OF BY-PRODUCTS
1. D 21.D
2. B 22.B
3. B 23.E
4. A 24.C
5. D 25.A
6. E 26.D
7. E 27.A
8. C 28.A
9. D 29.E
10.A 30.B
11.B 31.A
12.A 32.C
13.B 33.A
14.B 34.C
15.D 35.D
16.C 36.C
17.E 37.C
18.A 38.B
19.C 39.B
20.A 40.C
447
STANDARD COSTING
Computation of variances
1. B
Mix variance P450 U
Yield variance 150 U
Quantity variance P600 U
2. D
3. A
Total standard cost P72,000
÷ Std qty for actual production (14,400 x 4) 57,600
Standard price per unit of materials P1.25
The usage variance is P3,000 unfavorable. The standard price is P1.25. Using the
formula for Usage variance, the difference in quantity may be computed as
follows:
4. D
Actual price (P10,080 ÷ 4,200) P2.40
Standard price 2.50
448
Difference in prices – favourable P 0.10
X actual quantity purchased 4,200
Price variance – favourable P 420
5. C
Actual time – hours 4,100
Less standard time (1,000 x 4) 4,000
Difference in time – unfavorable 100
X standard rate per hour P 12
Efficiency variance – unfavorable P1,200
6. C
Controllable variance (P100 F + P400 U + P2,000 U) = 2,300 U
7. B
Spending variance (P100 F + P2,000 U) = P1,900 U
8. A
9. D
10.D
11.A
12.C
13.D
449
14.B
Var. cost (4,800*P66) P310,800
Fixed cost 80,000
Budgeted cost for actual production P396,800
15.C
Actual production 4,800
Multiply Std, cost per unit P 82
Total std. cost P393,600
16.B
Actual costs P400,000
Less total std. cost 393,600
Variance (Unfavorable) P 6,400
17.E
18.A
$87,000+[($87,000/$87,000+$348,000)-$12,500]*[($87,000/$87,000+
$348,000)-$15,000] = $86,500
19.C
$130,500+[($130,500/$130,500+$739,500)-$20,000]*[($130,500/$130,500+
$739,500)-$5,000] = $132,750
20.B
$348,000 + $739,500 + $591,600 + ($15,000 x .85) - ($2,500 x .80) - .85
($6,000) = $1,684,750
21.B
100,000 x $.78 = $78,000
22.D
8,000 x 4 x $3.60 = $115,200
23.A
26,000 - (8,000 x 3 x $1) = $2,000
24.C
The variance would be allocated only to finished goods and cost of goods sold.
25.A
(5,600 x $40) + (5,600 x $49.20) = $499,520
26.C
(800 x $40) + (800 x .75 x $49.20) = $61,520
27.C
$4,200 = $3.75 (x - 10,000)
$3.75 x = $4,200 + $37,500
450
x = 11,120
28.D
$240+.25(240)/30÷2 = $20/unit
29.B
Actual factory overhead $ 575,000
Budget allowance:
Variable factory overhead (52,000 x $6) $312,000
Budgeted fixed overhead 250,000 562,000
Controllable variance (unfavourable) $13,000
30.B
Budget allowance based on standard hours allowed
[(52,000 x $6) + $250,000] $ 562,000
Factory overhead applied at standard 572,000
Volume variance (favourable) $ (10,000)
31.C
Actual factory overhead $ 178,500
Budget allowance:
Variable for actual hours
(121,000 x $.50) $ 60,500
Fixed 110,000 170,500
Spending variance (unfavorable) $ 8,000
32.A
Budget allowance for actual hours
[(121,000 x $.50) + $110,000] $ 170,500
Budget allowance for standard hours:
Variable (130,000 x $.50) $ 65,000
Fixed 110,000 175,000
Variable efficiency variance (favourable) $ (4,500)
33.D
$3,000 = x (30,000 - 29,000)
1,000 x = $3,000
x = $3
y = $2.80 - $3.00(30,000)
y = ($6,000) favourable
34.A
x = $5.10 [7,800 - (2,000 x 4)]
x = ($1,020) favourable
451
35.D
x = $10 [2,000 - (900 x 2)]
x = $2,000 unfavorable
36.D
Variable budget allowance for actual hours (2,100 x $3) $ 6,300
Variable budget allowance for standard hours
($3 x 1,000 x 2) 6,000
$ 300
37.C
38.B
39.D
40.A
452
STANDARD COSTING
1. B 21.A
2. D 22.C
3. B 23.B
4. D 24.B
5. E 25.D
6. D 26.D
7. A 27.E
8. D 28.D
9. E 29.A
10.C 30.D
11.D 31.A
12.B 32.A
13.C 33.E
14.A 34.C
15.C 35.B
16.E 36.A
17.C 37.D
18.A 38.B
19.D 39.E
20.D 40.A
453
2,000
= 4 days
500